Sie sind auf Seite 1von 66

Sección 25

NEUROLOGIA

Autor
Dr. CARLOS MARTIN ESTEFANIA
Residente de Neurología
Hospital Clínico Universitario San Carlos
Madrid

Jefe de Servicio: Dr. E. Varela de Seijas Slocker


INDICE

NEUROLOGIA

Capítulo I. S INDROMES TOPOGRAFICOS Y Enfermedad de Parkinson


ALTERACIONES DEL LENGUAJE Atrofia multisistémica
Síndromes frontales Parálisis supranuclear progresiva
Síndromes temporales Enfermedad de Wilson
Síndromes parietales Enfermedad de Huntington
Síndromes occipitales Capítulo VI. INFECCIONES DEL SISTEMA NER-
Síndromes de desconexión VIOSO CENTRAL
Síndrome talámico
Síndromes del tronco cerebral Infecciones no víricas
Alteraciones del lenguaje Infecciones víricas

Capítulo II. COMA Capítulo VII. NEOPLASIAS DEL SISTEMA NER-


VIOSO CENTRAL
Definición
Clasificación y etiología Metástasis cerebrales
Enfoque clínico del coma Tumores primarios del SNC
Tratamiento Tumores de la médula espinal
Capítulo III. DEMENCIA Capítulo VIII. ENFERMEDAD DE LA NEURONA
MOTORA
Definición
Etiología Introducción
Clasificación Etiología
Formas clínicas Epidemiología
Enfermedad de Alzheimer Formas hereditarias de la enfermedad
Enfermedad de Pick (atrofia lobar) Características clínicas
Demencia vascular Exámenes complementarios
Hidrocefalia a presión normal (síndrome Tratamiento
Hakim-Adams)
Capítulo IX. ESCLEROSIS MULTIPLE
Complejo demencia-SIDA
Métodos de diagnóstico y tratamiento Enfermedades desmielinizantes: esclerosis
Seudodemencia múltiple
Esclerosis múltiple
Capítulo IV. ENFERMEDADES CEREBROVAS-
CULARES Capítulo X. EPILEPSIA
Patogenia Definición
Presentación clínica Clasificación de las crisis
Datos de laboratorio Etiología
Tratamiento Patogenia
Manejo de la tensión arterial en el accidente Cuadro clínico
cerebrovascular agudo Diagnóstico diferencial
Localización anatómica en los accidentes Tratamiento
cerebrovasculares Toxicidad de fármacos anticomiciales
Nuevos anticomiciales
Capítulo V. ENFERMEDAD DE PARKINSON Y
OTRAS ALTERACIONES DEL MOVIMIENTO Capítulo XI. CEFALEAS
Anatomía y fisiología de los ganglios basales Definición
INDICE

Anatomía
Migraña
Cefalea histamínica «en racimos» o de Hor-
ton
Cefalea de tipo tensional o de estrés
Clasificación y diagnóstico diferencial se-
gún curso temporal
Capítulo XII. PATOLOGIA MEDULAR
Anatomía
Semiología medular
Formas clínicas
Capítulo XIII. NEUROPATIAS PERIFERICAS
Polineuropatía
Mononeuropatía
Mononeuropatía múltiple
Capítulo XIV. SINDROMES NEUROCUTANEOS
CONGENITOS
Concepto
Neurofibromatosis (enfermedad de Von
Recklinghausen)
Esclerosis tuberosa (enfermedad de Bour-
neville)
Síndrome encéfalo-trigeminal (enfermedad
de Sturge-Weber)
Hemangioblastomatosis cerebelo-retiniana
(síndrome de Von Hippel-Lindau)
Capítulo XV. PATOLOGIA DE LA UNION NEU-
ROMUSCULAR
Trastornos autoinmunes
Trastornos congénitos
Trastornos tóxicos
Capítulo XVI. MIOPATIAS
Manifestaciones clínicas
Métodos diagnósticos
Clasificación de miopatías
Capítulo XVII. DEGENERACIONES ESPINOCE-
REBELOSAS
Concepto
Clasificación
Ataxia de Friederich
Paraplejía espástica hereditaria

BIBLIOGRAFIA
INDICE DE MATERIAS
Capítulo I

SINDROMES
T OPOGRAFICOS
Y ALTERACIONES
DEL LENGUAJE

Indice
Síndromes frontales Síndromes de desconexión
Síndromes temporales Síndrome talámico
Síndromes parietales Síndromes del tronco cerebral
Síndromes occipitales Alteraciones del lenguaje

Dr. JAVIER PARDO MORENO

SINDROMES FRONTALES Lesiones limitadas al área 6 producen una hipocinesia del


miembro superior contralateral con perseveración motora y
En los lóbulos frontales se distinguen habitualmente tres grasping contralateral (reflejo de prensión). Son frecuentes
grandes secciones con interés funcional, de atrás a delante: las alteraciones del tono, con oposición a la movilización
área prerrolándica o motora, área premotora y área prefrontal. pasiva, y varía con el grado de atención del paciente, fenó-
El área prerrolándica o motora está situada por delante del meno denominado paratonía o Gegenhalten. En lesiones bi-
surco central (área 4 de Brodmann). Las lesiones de esta área laterales se produce una apraxia de la marcha. Las lesiones
producen una parálisis de cara mano y pierna contralaterales, de- del área 8 interfieren con el mecanismo de la mirada lateral
sarrollando un síndrome de lesión de primera motoneurona. La hacia el lado contrario a la lesión, por lo que la mirada apa-
paresia afecta a los segmentos distales de los miembros. rece tónicamente desviada hacia el lado del hemisferio le-
El área premotora incluye las áreas 6 y 8 de Brodmann. La sionado. En el hemisferio izquierdo la lesión del área moto-
porción más alta y la correspondiente a la cara medial se de- ra suplementaria puede dar lugar a una afasia transcortical
nomina área motora suplementaria. motora.

1629
SINDROMES TOPOGRAFICOS Y ALTERACIONES DEL LENGUAJE

En el hemisferio dominante, en el pie de la 3.a circunvolu- térmicos, táctiles o dolorosos permanece relativamente respe-
ción frontal se encuentra el área de Broca (área 44), cuya le- tadas.
sión produce la afasia motora de Broca. En las lesiones más bajas puede afectarse la vía visual, con
El córtex prefrontal está situado por delante del área premo- cuadrantanopsia inferior homónima contralateral, generalmen-
tora. Sus signos cardinales son: pérdida de iniciativa y espon- te poco congruente.
taneidad, con tendencia a la inactividad motora; cambios en la
personalidad con despreocupación, desinhibición, estado de- Negligencia y trastornos afines
nominado moria frontal; deterioro cognitivo, manifestado como
Negligencia : Es la incapacidad para detectar, atender u
alteración de la concentración, pérdida de la abstracción tem-
orientar un estímulo significativo presentado a través del he-
poral, empobrecimiento del juicio, con tendencia a asociacio-
nes irrelevantes y concretas. micuerpo contralateral, siempre que la alteración no pueda
atribuirse a déficit sensitivo o motor.
La asomatognosia: Es una forma especial de negligencia ha-
SINDROMES TEMPORALES cia el propio hemicuerpo.
Cuando el paciente ignora su propio déficit neurológico se
Alteraciones sensoriales denomina anosognosia.
— Alucinaciones olfatorias por lesión de la porción ante- La extinción sensitiva consiste en que al presentar un estí-
rior e inferomedial del lóbulo temporal. mulo bilateral simultáneo, ignora o no reconoce el estímulo
— Sordera cortical por lesión bilateral de las áreas audi- contralateral a la lesión, aunque percibe perfectamente cada
tivas primarias. estímulo separado.
— Ilusiones y alucinaciones auditivas que pueden ser
elementales (murmullos, pitos) o complejas (palabras, Trastornos visuespaciales
temas musicales). Son más evidentes en las lesiones del parietal no dominante.
— Agnosias auditivas por lesión de las áreas de asocia-
ción auditiva. Síndrome de Gerstmann
— Cuadrantanopsia homónima superior contralateral a la
lesión por afectar a las fibras más inferiores de las ra- Lesión parietal posterior del hemisferio dominante, clásica-
diaciones que rodean el asta temporal del ventrículo mente en la región del gyrus angularis y consiste en agrafía,
(asta de Meyer). acalculia, agnosia digital y confusión derecha-izquierda.
— Ilusiones y alucinaciones visuales de carácter comple-
jo por lesión de la encrucijada témporo-occipital. SINDROMES OCCIPITALES

Percepción del tiempo y la memoria Defectos campimétricos y ceguera cortical


Las distorsiones en la percepción del tiempo pueden ocurrir El déficit campimétrico más frecuente por lesión del lóbulo
con lesiones tanto derechas como izquierdas. occipital es una hemianopsia homónima contralateral.
La amnesia que se produce por lesiones temporales bilate- Cuando existe lesión bilateral se produce el síndrome de ce-
rales en su porción anterior es una amnesia de fijación como la guera cortical, que se caracteriza por conservación del reflejo
que se ve en el Síndrome de Korsakoff. fotomotor, desaparición del ritmo alfa del EEG y los potencia-
Las crisis del lóbulo temporal pueden manifestarse como les evocados visuales están abolidos. Con frecuencia el pa-
alucinaciones experienciales, siendo el más frecuente el fenó- ciente presenta anosognosia y esta peculiar forma de anosog-
meno de dejà vu (ya visto). nosia en la ceguera cortical constituye el síndrome de Antón.

Emociones y conducta Ilusiones y alucinaciones visuales


La lesión unilateral del lóbulo temporal, en su parte anterior, Las lesiones occipitales pueden presentarse como distorsio-
se asocia con cambios conductuales de tipo paranoide, depre- nes de la percepción que se producen como consecuencia de
sivo o psicopático con agresividad. la afección de la encrucijada parieto-témporo-occipital.
Las alucinaciones suelen ser menos elaboradas que las que
SINDROMES PARIETALES aparecen en las lesiones de la porción posterior del lóbulo
temporal.
Síndrome sensitivo cortical
Síndrome de Balint
La lesión de la circunvolución poscentral produce una pérdi-
da de la capacidad de localizar estímulos táctiles, de distinguir Se asocia a las lesiones bilaterales en la región parieto-oc-
la forma de los objetos (astereognosia), o la discriminación en- cipital. Cursa con parálisis psíquica de la mirada o incapacidad
tre dos puntos; mientras que la percepción de los estímulos para dirigir la mirada voluntariamente a pesar de que los movi-

1630
NEUROLOGIA

mientos oculares son normales; ataxia óptica; inatención vi-


25
sual. 1
¿Cómo se denomina al síndrome que cursa con agrafía, agnosia digital,
SINDROMES DE DESCONEXION confusión derecha-izquierda y acalculia?:

Tras la sección completa del cuerpo calloso las áreas visua- 1. Sd. de Balint.
2. Sd. de Gerstmann.
les y del lenguaje izquierdas quedan aisladas del hemisferio 3. Sd. de Anton.
derecho, por lo que no pueden nombrar con los ojos cerrados 4. Sd. de Korsakoff.
un objeto situado en su mano izquierda, no ejecutarían correc- 5. Sd. de Reye.
tamente una orden verbal con su mano izquierda y tendrán una
anomia en el campo visual izquierdo.

SINDROME TALAMICO 2
Síndrome sensitivo talámico La lesión aérea 8 de Brodmann se caracteriza por:

Una lesión talámica generalmente produce una pérdida de 1. Lesión del III par craneal.
2. Ataxia óptica.
todas las modalidades sensitivas que afecta al hemicuerpo 3. Desviación tónica de la mirada hacia el hemisferio lesionado.
contralateral, incluyendo cara. 4. Ceguera cortical.
5. Anosognosia.
Afasia subcortical
Se caracteriza por mutismo en la fase inicial que luego da
paso a un lenguaje hipofónico, con fluencia disminuida y multi-
tud de parafasias, con repetición conservada. 3
El fenómeno de dejà vu aparece en lesiones:
SINDROMES DEL TRONCO CEREBRAL
1. Lóbulo temporal.
2. Lóbulo parietal.
Las lesiones del tronco cerebral pueden localizarse siguien-
3. Lóbulo occipital.
do unos principios basados en el conocimiento anatómico de 4. Lóbulo frontal.
su estructura. 5. Tronco cerebral.
La presencia de alteraciones en los pares craneales sirve de
guía de la altura de la lesión del meséncefalo (III par, mirada
vertical), protuberancia (VI, VII, mirada lateral) o bulbo (pares
IX, X, XII, núcleo sensitivo del V).
4
Las lesiones de situación anterior cursan con afectación pi-
ramidal protagonista, en tanto que las lesiones posteriores El tipo de afasia más frecuente es:
respetan la vía piramidal y producen lesión de vías sensitivas y 1. Afasia de Broca.
cerebelosas sin paresia. 2. Afasia anómica.
3. Afasia de Wernicke.
ALTERACIONES DEL LENGUAJE 4. Afasia global.
5. Afasia transcortical.
Se entiende por afasia aquel trastorno adquirido del lengua-
je, debido a una lesión cerebral y que se caracteriza por pre-
sentar al menos una de las siguientes características: errores
en lenguaje escrito (parafasias), alteración de la comprensión, 5
dificultad para encontrar palabras (anomia) o alteración en la La paratonía es típica de:
fluidez del lenguaje.
Para valorar una afasia hay que tener en cuenta: fluencia 1. Síndromes parietales.
del lenguaje hablado, capacidad de repetición, grado de com- 2. Síndromes occipitales.
3. Síndromes temporales.
prensión, nominación, lectura y escritura.
4. Síndromes de desconexión.
Los distintos tipos de afasia son: 5. Síndromes frontales.

— Afasia de Broca: lesión en el área 44 de Brodmann. El


habla es no fluente, disprosódica, agramatical, disár- RESPUESTAS: 1: 2; 2: 3; 3: 1; 4: 2; 5: 5.
trica con la comprensión relativamente preservada y

1631
SINDROMES TOPOGRAFICOS Y ALTERACIONES DEL LENGUAJE

la repetición, nominación, lectura y escritura altera- — Afasias transcorticales: la lesión respeta estructuras
das. perisilvianas incluyendo el área de Broca y Wernicke.
— Afasia de Wernicke: lesión en la porción posterior de Se caracterizan por tener indemne la repetición. Pue-
la circunvolución temporal superior del hemisferio do- den ser motoras, sensitivas o mixtas y sus caracterís-
minante. ticas son superponibles a las afasias de Broca, Wer-
Lo que domina el cuadro es fundamentalmente una in- nicke o global respectivamente, salvo que ninguna de
capacidad para comprender ninguna orden. El habla estas últimas puede repetir.
es fluente o hiperfluente con un lenguaje vacío de — Afasia global: es superponible a la afasia de Wernic-
contenido, contaminado con abundantes parafasias y ke, pero además el habla es poco fluente.
neologismos constituyendo la denominada jergafasia. — Afasia anómica: es el tipo de afasia más frecuente, ya
No pueden repetir, leer, nominar ni escribir. que suele ser el defecto residual tras la recuperación
— Afasia de conducción: lesión en estructuras perisilvia- de alguno de los anteriores tipos de afasia. Presenta
nas respetando el área de Broca y Wernicke. El habla es una alteración fundamentalmente de la nominación
fluente y la comprensión está relativamente preservada. teniendo el resto de las funciones relativamente pre-
La nominación y lectura están relativamente afectadas y servadas (fluencia, comprensión, repetición, lectura y
la escritura y la repetición están muy alteradas. escritura).

1632
Capítulo II

COMA

Indice

Definición Enfoque clínico del coma


Clasificación y etiología Tratamiento

DEFINICION — Intoxicación exógena (30%).


— Encefalopatía hipóxica.
Falta de respuesta del paciente frente a estímulos. — Otras causas metabólicas.
— A.C.V. hemorrágico.
Estupor — A.C.V. isquémico.
— Hematoma subdural.
Estado en que se puede despertar al paciente mediante es- — Meningoencefalitis.
tímulos enérgicos, las respuestas verbales son mínimas o nu- — Hemorragia subaracnoidea.
las y el enfermo realiza algún esfuerzo para evitar los estímu- — Tumores.
los dolorosos. — Causas psiquiátricas.
— Otras.
Coma
ENFOQUE CLINICO DEL COMA
Estado en el que el paciente no puede ser despertado me-
diante estimulación y en el que el enfermo no hace esfuerzos Historia clínica
para evitar los estímulos molestos.
Existen casos en que la etiología es clara: traumatismos, pa-
rada cardíaca, etc.
CLASIFICACION Y ETIOLOGIA
Es importante conocer la forma de inicio: súbita, subaguda,
crónica. Reseñar los antecedentes personales: diabetes, hiper-
El coma puede dividirse en dos bloques fundamentales:
tensión, cardiopatía, hepatopatía, nefropatía, enfermedad res-
piratoria, enfermedad neurológica, alcoholismo, drogadicción,
— Comas de origen metabólico, los más frecuentes.
ingesta de fármacos...
— Comas de origen estructural. A estos dos grupos po-
dríamos añadir los de origen psicógeno. Exploración física
Siguiendo la clasificación de Plum y Posner podemos obser- Debe realizarse una exploración física general y una explo-
var las etiologías más frecuentes. ración específica del coma que comprende los siguientes apar-

1633
COMA

tados: nivel de conciencia, patrón respiratorio, pupilas, movi- — Nistagmo de convergencia, indica lesión mesencefáli-
mientos oculares y respuesta motora. ca.

Nivel de conciencia Posición primaria de la mirada


Describir la situación del enfermo, su reactividad ante dife- La existencia de una desviación conjugada suele ser debida
rentes estímulos. Puede usarse la escala de Glasgow: a una lesión focal, una desviación hacia hemicuerpo parético
Usada en principio para los traumatismos cráneo-encefáli- indica una lesión deficitaria de tronco ipslateral o una lesión
cos, pero últil en la valoración del nivel de conciencia de cual- irritativa en hemisferio cerebral contralateral. Una desviación
quier sujeto: de la mirada hacia el hemicuerpo sano es signo de lesión en el
hemisferio observado.
Respuesta verbal: Orientada. La desconjugación no es exclusiva de lesión estructural.
Confusa.
Palabras inapropiadas. Reflejos óculo-cefálicos y óculo-vestibulares
Sonidos imperceptibles. (ROC, ROV)
Nada.
Apertura ocular: Espontánea. Su normalidad indica integridad del tronco.
Al hablar el explorador. ROC, al girar bruscamente la cabeza los ojos se mueven de
Al dolor. forma conjugada en dirección contraria (esto no ocurre cuando
Ninguna. existe afectación del tronco ni en pacientes vigiles).
Respuesta motora: Ante estímulos dolorosos. ROV, 50 cc. de agua fría son instilados por el conducto audi-
Localiza estímulos. tivo externo (cabeza a 30 grados) produciéndose una desvia-
Retira miembros. ción tónica y conjugada de los ojos hacia el oído estimulado,
Flexiona extremidades. en el vigil se produce un nistagmo contralateral.
Extiende extremidades. Nistagmo «huye del frío». Si se instala agua caliente las di-
Ninguna. recciones del nistagmo eran las contrarias.

Patrón respiratorio Respuesta motora

— Respiración de Cheyne-Stokes.Típica de afectación Hay que valorar la motilidad espontánea y la motilidad an-
diencefálica o bihemisférica. te estímulos, observando si la respuesta es apropiada, anor-
— Hiperventilación. Afectación mesencefálica. mal o inapropiada (posturas de decorticación y descerebra-
— Respiración apneústica. Por alteración protuberancial. ción)
— Respiración en cúmulos. Tambien en afectación protu-
berancial. — Postura de decorticación: daño bihemisférico.
— Respiración atáxica de Biot. Lesión bulbar. Brazo flexionado, adducido y supinado. Muñecas fle-
xionadas.
Pierna extendida, rotación interna y pie en flexión
Pupilas
plantar.
Hay que valorar el tamaño, la simetría y reactividad a la luz. — Postura de descerebración: daño mesencefálico o pro-
— Medias y poco reactivas. Daño diencefálico. tuberancial.
— Midriáticas y fijas. Lesión mesencefálica (o por anti- Brazo extendido, adducido y pronado.
colinérgicos). Pierna extendida y en rotación interna.
— Puntiformes y fijas. Lesión en protuberancia (opiá-
ceos). Pruebas diagnósticas complementarias
— Anisocóricas: midriática reactiva. Afección simpática Glucemia y EKG son las dos primeras determinaciones, pos-
homolateral. teriormente será necesario hemograma, creatinina, electróli-
— Midriática fija. Herniación uncal. tos, gasometría arterial, sedimento urinario y placa de tórax.
Según los hallazgos clínicos y la sospecha diagnóstica podre-
Movimientos oculares mos solicitar: TAC craneal, EEG, LCR, niveles de fármacos y tó-
xicos.
Movimientos oculares espontáneos
— Robbing, movimientos horizontales erráticos, asegu- TRATAMIENTO
ran que el tronco está intacto.
— Bobbing, movimientos conjugados hacia abajo, indi- El tratamiento fundamental es el etiológico, pero deberán
can lesión de la protuberancia. realizarse unas medidas de soporte inmediato:

1634
NEUROLOGIA

— Asegurar vía respiratoria libre.


25
— Asegurar vía venosa permeable. 6
— Medidas farmacológicas: Ante un cuadro de intoxicación por opiáceos el tratamiento de elección
es:
• Sospecha de intoxicación por opiáceos: Naloxona
i.v. 1. Flumacenil.
2. Corticoides.
• Hipoglucemia: administrar glucosa .
3. Naloxona.
• Sospecha de etilismo o encefalopatía de Werni- 4. Inmunoglobulinas.
ke: tiamina. 5. Plasmaféresis.
• Intoxicación por benzodiacepinas: flumacenil.

Habrá que evitar la administración de sedantes y de oftal- 7


mopléjicos.
En la valoración clínica del coma son importantes los siguientes datos ex-
cepto:
1. Respuesta motora.
2. Motilidad ocular.
3. Alteraciones sensitivas.
4. Patrón respiratorio.
5. Pupilas.

8
En las lesiones protuberannciales cabe esperar cualquiera de las siguien-
tes manifestaciones excepto:
1. Pupilas midriáticas.
2. Parálisis facial ipsilateral.
3. Hemiparesia ipsilateral.
4. Alteraciones de la motilidad ocular conjugada.
5. Hemihipoestesia contralateral.

9
De las siguientes etiologías, la más frecuente en la etiología del coma
es:
1. Ictus hemorrágico.
2. Meningitis bacteriana.
3. Intoxicaciones.
4. Tumores cerebrales.
5. Crisis comiciales.

10
¿Cuál sería la sospecha ante un paciente con bajo nivel de conciencia,
hemiplejía izquierda con Babinski izquierdo, desviación de la mira-
da conjugada hacia la derecha y pupila izquierda midriática y fi-
ja?:
1. Infarto protuberancial izquierdo.
2. Intoxicación por opiáceos.
3. Brote de esclerosis múltiple.
4. Infarto hemisférico derecho con herniación uncal.
5. Infarto troncoencefálico bilateral en evolución.

RESPUESTAS: 6: 3; 7: 3; 8: 1; 9: 3; 10: 4.

1635
Capítulo III

DEMENCIA

Indice
Definición Demencia vascular
Etiología Hidrocefalia a presión normal (síndrome Hakim-
Clasificación Adams)
Formas clínicas Complejo demencia-SIDA
Enfermedad de Alzheimer Métodos de diagnóstico y tratamiento
Enfermedad de Pick (atrofia lobar) Seudodemencia

DEFINICION — Enfermedad de Alzheimer.


— Demencia mixta (Alzheimer + vascular).
Alteración adquirida de la función intelectual, que cursa con — Demencia multi-infarto.
deterioro del intelecto, memoria y personalidad, en presencia — Demencia alcohólica.
de un nivel de conciencia normal. La presencia de un nivel de — Hidrocefalia a presión normal.
conciencia normal lo diferencia del cuadro confusional agudo. — Tumores.
El deterioro intelectual puede afectar a todas las funciones — Otras: demencias metabólicas, enfermedad de Parkin-
intelectuales, aunque en períodos iniciales suele afectar a las son, etc.
siguientes áreas: lenguaje o memoria, destreza visoespacial,
afecto o personalidad. CLASIFICACION
Los criterios fijados por el DSM-III(r) para el diagnóstico de
demencia son: — Demencias primarias: enfermedad de Alzheimer, en-
fermedad de Pick.
— Desadaptación sociolaboral. — Demencias en relación con procesos vasculares: de-
— Trastorno adquirido. mencia multi-infarto, infarto talámico, vasculitis, enf.
— Conservación del nivel de conciencia. Binswanger.
— Trastorno persistente (semanas, meses). — Demencias relacionadas con procesos infecciosos:
— Afectación de funciones superiores. VIH, sífilis, LMP, PEES, Creutzfeld-Jacob, Whipple,
TBC, hongos, protozoos.
ETIOLOGIA — Demencias en el contexto de otros procesos neuroló-
gicos: lesiones ocupantes de espacio, enfermedades
Por orden de frecuencia: con trastornos del movimiento (Parkinson, Wilson..).

1636
NEUROLOGIA

— Demencias en trastornos endocrinos y metabólicos:


25
hipotiroidismo, déficit de B12 y ácido fólico, hipogluce- 11
mia crónica, etc. Una de las siguientes entidades se engloba dentro del concepto de de-
— Demencias en trastornos psiquiátricos: esquizofrenia, mencias potencialmente tratables:
seudodemencia.
1. Enfermedad de Alzheimer.
2. Enfermedad de Whipple.
Desde un punto de vista práctico es interesante dividir a las 3. Leucoencefalopatía multifocal progresiva.
demencias en procesos potencialmente curables o no. Así, se 4. Enfermedad de Creutzfeldt-Jacob.
consideran irreversibles los siguientes procesos: Alzheimer, 5. Enfermedad de Marchiafava-Bignami.
Pick, Parkinson, Huntington, esclerosis múltiple, Creutzfeld-
Jacob, demencia-VIH, Leucoencefalopatía multifocal progresiva, 12
panencefalitis esclerosante subaguda, las demencias debidas al
alcoholismo: Marchiafava-Bignami y Wernicke-Korsakoff. Al practicar la derivación ventrículo-peritoneal en el tratamiento de la hi-
drocefalia a presión normal el dato espera que sufra mayor mejoría
es:
FORMAS CLINICAS
1. Trastorno de la marcha.
Desde un punto de vista clínico (y con su correspondiente 2. Incontinencia urinaria.
substrato anatómico) diferenciamos entre demencia cortical y 3. Deterioro cognitivo.
subcortical. 4. Alteraciones del lenguaje.
5. Trastornos del comportamiento.
Demencias corticales
13
Lo que predomina es una pérdida de las funciones relacio-
Las demencias con un patrón de afectación subcortical se caracterizan
nadas con el lenguaje, la percepción y el cálculo, junto con una
por los siguientes hallazgos excepto:
alteración de la memoria que es común a los dos tipos. Es la
demencia de la enfermedad de Alzheimer. 1. Alteraciones del comportamiento.
2. Cuadros afásicos.
Demencias subcorticales 3. Insomnio.
4. Signos extrapiramidales.
Se caracterizan por afectación del nivel afectivo, trastornos 5. Alteraciones del estado de ánimo.
del estado de ánimo y del nivel de vigilia. Anatómicamente se
produce afectación de ganglios basales, tálamo y sustancia 14
blanca profunda. Es típico de los trastornos extrapiramidales o Paciente varón de 75 años de edad que acude a consultas externas de
de la hidrocefalia a presión normal. neurología por un cuadro de 2 años de evolución consistente en tor-
peza de la marcha, apatía, dificultad para manejar el dinero, algún
ENFERMEDAD DE ALZHEIMER cuadro de desorientación (incluso en su propio domicilio) y en los úl-
timos meses incontinencia esfinteriana. Su diagnóstico de sospecha
Causa más frecuente de demencia en el anciano. Lo pade- sería:
cen hasta un 47% de las personas mayores de 85 años. 1. Enfermedad de Pick.
2. Hidrocefalia a presión normal.
Etiología 3. Enfermedad de Creutzfeld-Jacob.
4. Enfermedad de Alzheimer.
— La mayor parte de los casos son esporádicos, un 10% 5. Demencia multiinfarto.
familiar (A.D.).
— Alteraciones polimórficas en el brazo largo del cromo-
soma 21. En la trisomía 21 existen lesiones idénticas 15
a las del Alzheimer. ¿Cuál de los siguientes datos no le sugiere un cuadro de seudodemen-
— Otros factores: traumatismos previos, factores tóxi- cia?:
cos, infecciosos, inmunológicos, vasculares. 1. Inicio súbito.
2. Antecedentes psiquiátricos.
Anatomía patológica 3. Trastornos del sueño.
4. Poca preocupación por su enfermedad.
— Destrucción y desaparición de neuronas en la corteza ce-
5. Amnesia muy llamativa.
rebral, que condiciona atrofia importante de las circunvo-
luciones, sobre todo en regiones frontal y parietal.
RESPUESTAS: 11: 2; 12: 1; 13: 2; 14: 2; 15: 4.
Microscópicamente:

1637
DEMENCIA

— Placas seniles. Situadas en corteza cerebral e hipo- DEMENCIA VASCULAR


campo en el espacio extracelular.
— Degeneración neurofibrilar. Viene definida por:
— Gliosis y despoblación neuronal.
— Angiopatía amiloide. — Demencia.
— Deterioro escalonado con distribución precoz y parchea-
Bioquímica da de los déficit y síntomas neurológicos focales.
— Evidencia de enfermedad cerebrovascular etiológica-
— Déficit colinérgico: disminución de la enzima acetilco- mente relacionada con el cuadro demencial.
lin transferasa en córtex y en hipocampo.
— Déficit de noradrenalina, serotonina, somatostatina y Hachinsky ha confeccionado una escala para la valoración de
neuronas peptidérgicas. la demencia vascular valorando: inicio agudo (2), deterioro a bro-
tes (1), curso fluctuante (2), confusión nocturna (1), conservación
Cuadro clínico de la personalidad (1),depresión (1), síntomas somáticos (1), labili-
Las alteraciones clínicas siguientes son las típicas de la de- dad emocional (1), historia de ACV (1), historia de HTA (1), sínto-
mencia cortical, donde se producen alteraciones de: mas neurológicos focales (2), y otros signos de aterosclerosis (1).
Así, una demencia con más de 7 puntos y sin otra causa evi-
— Función intelectual: alteración de capacidad de abs- dente puede considerarse de origen vascular.
tracción.
— Memoria: afectación de la memoria a largo y medio Formas clínicas
plazo. Demencia vascular cortical o demencia multi-infarto
— Lenguaje: empobrecimiento semántico.
Demencia producida por sumación de infartos corticales, que
— Praxias-Gnosias: 1.°-apraxia constructiva, posterior-
pueden ser subclínicos. Puede asociarse a cualquier tipo de en-
mente apraxia del vestido.
fermedad cerebral vascular, aunque con más frecuencia es por
— Comportamiento: negación de la evidencia de enfer-
medad, se asocian cuadros depresivos (más frecuente embolia bilateral recidivante originada en corazón o en las arte-
en demencia vascular). rias carótidas. También puede deberse a procesos vasculíticos.

Demencia vascular subcortical


Tratamiento
— Estado lacunar. Infartos pequeños y múltiples en zo-
— Apoyo familiar.
nas subcorticales por hipertensión.
— Laborterapia.
— Enf. Binswanger. La enfermedad hipertensiva o ate-
— Control de patología intercurrente: infección...
rosclerótica produce engrosamiento de las paredes de
— Desde 1996 aprobada la tacrina, un anticolinesterásico
que podría disminuir la progresión de la enfermedad. los vasos perforantes profundos y de los capilares que
irrigan la sustancia blanca subcortical.
ENFERMEDAD DE PICK (ATROFIA LOBAR)
HIDROCEFALIA A PRESION NORMAL
Se produce una atrofia cerebral circunscrita. Se debe incluir (SINDROME HAKIM-ADAMS)
en el diagnóstico diferencial de demencia en el período presenil.
Frecuente su transmisión hereditaria (dominante) y más fre- Síndrome de hidrocefalia comunicante en el que no existe o
cuente en mujeres. no podemos demostrar hipertensión intracraneal.
Se produce una alteración en la dinámica del LCR que no
Anatomía patológica llega de forma adecuada a los plexos coroideos y por tanto
tiene que reabsorberse a través del epéndimo, lo que provoca una
Atrofia intensa de porciones anteriores de lóbulos frontal y alteración secundaria de la sustancia blanca periventricular.
temporal.
Microscópicamente depósitos fibrilares en citoplasma, son Etiología
masas de filamentos rectos, a diferencia del Alzheimer, en que
son helicoidales. En un 50% de los casos desconocida, pueden asociarse an-
En algunas neuronas, cuerpos de Pick. tecedentes de una hemorragia subaracnoidea o un proceso in-
flamatorio meníngeo basal.
Cuadro clínico
Cuadro clínico
Son características las manifestaciones que indican déficit
de los lóbulos frontal y temporal. Tríada típica: Demencia+alteración de la marcha de tipo ata-
Progresión lenta y constante. Duración media de 7 años. xia sin alteraciones cerebelosas+incontinencia esfinteriana.

1638
NEUROLOGIA

Puede realizarse diagnóstico ex-iubantibus si observamos


25
mejoría al realizar punción lumbar y extraer 15-20 cc. 16
Son datos anatomopatológicos típicos de la enfermedad de Alzheimer ex-
Tratamiento cepto:
Punciones lumbares de repetición y asociar acetazolamida. 1. Placas seniles.
Si la presión del LCR es mayor de 100 mmHg. puede instaurar- 2. Angiopatía amiloide.
se tratamiento quirúrgico. 3. Hipopigmentación de sustancia nigra.
En caso de ser efectiva la derivación ventrículo-peritoneal lo 4. Gliosis neuronal.
es en la mejoría de la marcha pero no en el deterioro cognitivo. 5. Degeneración neurofibrilar.

COMPLEJO DEMENCIA-SIDA o también


denominado trastorno cognitivo-motor asociado
a la infección por VIH 17
Se ha relacionado la enfermedad de Alzheimer con:
Aparece en un 6% de pacientes con SIDA, en relación con
afectación cerebral por infección directa por el virus. Es una 1. Factores de riesgo vascular.
demencia subcortical, progresiva de evolución rápida (semanas 2. Alteraciones en el cromosoma 21.
o meses), con alteraciones de la marcha, esfinterianas, mioclo- 3. Aumento de la presión intracraneal.
4. Atrofia cerebral sucortical.
nías, apatía y desinterés.
5. Defectos vitamínicos.
METODOS DE DIAGNOSTICO Y TRATAMIENTO

— Realizar historia clínica precisa.


— Pruebas complementarias. Teniendo en cuenta la 18
existencia de demencias potencialmente tratables hay La demencia multiinfarto se asocia de manera más constante con:
que realizar un screening solicitando: sistemático de 1. Embolias bilaterales recidivantes.
sangre con velocidad de sedimentación, glucemia, 2. Hipertensión arterial.
equilibrio hidroelectrolítico, pruebas básicas de fun- 3. Vasculitis.
ción hepática y tiroidea, vitamina B12 y ácido fólico, 4. Oclusión carotídea.
serologia luética, radiología de tórax, EKG y TAC cra- 5. Infartos subcorticales.
neal. Tambien se puede solicitar estudio de LCR, sero-
logía, estudio inmunitario, niveles de cobre y cerulo-
plasmina...
19
En cuanto al tratamiento de aquellas causas de demencia Es típico de la demencia vascular, excepto:
irreversibles será de apoyo al paciente y a la familia.
1. Curso clínico escalonado.
SEUDODEMENCIA 2. Asociación a hipertensión arterial.
3. Aparición de cuadro afaso-apráxico.
4. Fluctuación de la clínica.
Causas no orgánicas de deterioro intelectual. 5. Aparición de trastornos de la memoria.

— Pacientes con depresión grave aparecen desmotiva-


dos y con deterioro de funciones intelectuales. El
diagnóstico se sugiere por un inicio súbito, trastornos
del sueño, antecedentes psiquiátricos, acontecimiento 20
personal precipitante. Los deprimidos presentan ma- La hidrocefalia a presión normal se relaciona con:
yor afectación intelectual de la que indican los tests.
Se quejan de su deterioro y presentan variabilidad en 1. Descenso de la presión intracraneal.
2. Antecedentes de meningitis.
su afectación cuando se les estudia varias veces.
3. Traumatismos.
— Los histéricos presentan una amnesia muy llamativa. 4. Factores de riesgo vascular.
— Síndrome de Ganser, los pacientes responden con res- 5. Factores hereditarios.
puestas aproximadas. Responden de manera equivo-
cada a órdenes sencillas, pero sus contestaciones
siempre tienen algo que ver con el tema requerido. RESPUESTAS: 16: 3; 17: 2; 18: 1; 19: 3; 20: 2.

1639
Capítulo IV

ENFERMEDADES
CEREBROVASCULARES

Indice
Patogenia Manejo de la tensión arterial en el accidente cere-
Presentación clínica brovascular agudo
Datos de laboratorio Localización anatómica en los accidentes cerebro-
Tratamiento vasculares

Dr. JAVIER PARDO MORENO

PATOGENIA — Edema cerebral.


— Aumento del hematoma intracraneal por continuación
El proceso puede ser intrínseco al vaso (arteriosclerosis, li- de la hemorragia.
pohialinosis, inflamación, depósito de amiloide, disección ar- — Embolos que se propagan, se lisan y se dispersan.
terial, malformación, dilatación aneurismática o trombosis ve- — Embolia recurrente de arteria a arteria.
nosa); originarse a distancia (un émbolo del corazón o proce-
dente de la circulación extracraneal se aloja en un vaso intra- Accidente cerebrovascular establecido
craneal); ser resultado de disminución de la presión de perfu- Déficit neurológico que no progresa a lo largo de una obser-
sión o de aumento de la viscosidad sanguínea con flujo sanguí- vación de varios días.
neo inadecuado; producirse a partir de la ruptura de un vaso en
el espacio subaracnoideo o en el tejido intracraneal. Accidente isquémico transitorio (TIA)
Déficit neurológico focal que se resuelve de forma comple-
PRESENTACION CLINICA ta dentro de las primeras 24 horas. Un episodio único de défi-
cit neurológico focal que dura más de 30 minutos pero menos
Accidente cerebrovascular en evolución de 24 horas sugiere un TIA embólico.
Déficit neurológico que progresa o fluctúa mientras está en
observación. Se puede deber a: Ictus isquémico
Déficit neurológico producido por la interrupción de flujo
— Trombo que se propaga obliterando las ramas colate- sanguíneo a una región del cerebro debido a oclusión intravas-
rales, aumentando el territorio cerebral isquémico. cular o a un estado de bajo flujo. El déficit concuerda con el te-
— Estrechamiento progresivo de un vaso por un trombo. rritorio vascular irrigado por el vaso.

1640
NEUROLOGIA

Ictus embólico
25
Aparición brusca de un déficit que alcanza su grado máximo 21
desde el comienzo. En un infarto de A. cerebral media no se suele ver:
1. Hemiparesia.
Hemorragia intracraneal 2. Afasia.
3. Hemianopsia homónima.
Las formas más frecuentes son la hemorragia intracraneal 4. Ataxia.
hipertensiva y la lobular (50%), la rotura de un aneurisma sa- 5. Desviación tónica de la mirada hacia el lado de la lesión.
cular y la rotura de una malformación AV. En casi todos los ca-
sos se producen vómitos y en la mitad, cefalea.
La hemorragia hipertensiva típicamente ocurre en: el puta-
men, zona contigua de la cápsula interna y sustancia blanca
cerebral; tálamo; protuberancia; cerebelo. 22
Los signos oculares son importantes en la localización de La endarterectomía quirúrgica se debe realizar:
estas hemorragias: en el putamen, ojos desviados hacia el la- 1. En los ictus cardioembólicos.
do contrario de la parálisis (hacia la lesión); en el tálamo, ojos 2. En los infartos de A. cerebral posterior.
desviados hacia abajo, a veces con pupilas arreactivas; ponti- 3. En las hemorragias subaracnoideas.
nas, alteración de los movimientos oculares laterales reflejos y 4. En estenosis carotídeas > 70% sintomáticas
pupilas pequeñas (1-2 mm.) y reactivas. 5. En los ictus lacunares.

Ictus lacunar
Infarto que puede situarse en una localización anatómica de
0,5-1,0 cm. de diámetro, que resulta de enfermedad local de 23
pequeños vasos. Con respecto a la TAC craneal (señale la falsa):
1. Puede excluir hemorragia.
DATOS DE LABORATORIO 2. Es el método más útil para visualizar la fosa posterior.
3. Puede ser normal en infartos de corta evolución.
TAC craneal 4. No suele detectar lesiones en troncoencéfalo.
5. Puede detectar el edema que rodea al infarto.
Sin contraste, excluye de forma inmediata la hemorragia co-
mo la causa de accidente cerebrovascular focal, puede detec-
tar el edema que lo rodea y, con menos fiabilidad, un infarto
hemorrágico. No puede detectar la mayor parte de los infartos 24
al menos en las primeras 48 horas y a menudo no detecta le-
siones en la corteza o en el tronco del encéfalo. El tipo de hemorragia parenquimatosa más frecuente es:
1. De etiología hipertensiva.
RM 2. Por ruptura de un aneurisma.
3. Por ruptura de una MAV.
Los infartos se pueden ver en horas, incluso los que se en- 4. Tras ingesta de fármacos.
cuentran en la corteza cerebral y fosa posterior, así como las 5. Por anticoagulación.
lagunas <0,5 cm.

Exploración no invasiva de las carótidas


ECO-doppler de troncos supraaórticos para evidenciar este- 25
nosis arteriales. Los infartos mesencefálicos pueden cursar con (señale la falsa):

Angiografía cerebral 1. Afasia.


2. Paresia del III par.
Por caracterización retrógrada de la arteria femoral 3. Paresia de la mirada vertical.
4. Nistagmo de convergencia.
ECG, ecocardiografía y monitorización con Holter 5. Desorientación.
de 24 horas
Para evaluación en casos de sospecha de accidente cerebro- RESPUESTAS: 21: 4; 22: 4; 23: 2; 24: 1; 25: 1.
vascular embólico/TIA.

1641
ENFERMEDADES CEREBROVASCULARES

Estudios de coagulación LOCALIZACION ANATOMICA EN


LOS ACCIDENTES CEREBROVASCULARES1
Fundamentalmente el TP y TTPA, en particular en pacientes
que reciben tratamiento anticoagulante, así como en la hemo-
Hemisferio cerebral, cara lateral (a. cerebral media)
rragia cerebral.
Hemiparesia, déficit hemisensitivo, afasia, hemianopsia ho-
TRATAMIENTO mónima o cuadrantanopsia, mirada preferentemente desviada
hacia el lado de la lesión.
Ictus embólico o TIA
Anticoagulación Hemisferio cerebral, cara medial
(a. cerebral anterior)
La heparina IV se utiliza cuando se sospecha una oclusión
vascular aterotrombótica en evolución o circulación posterior. Parálisis del pie y de la pierna con o sin paresia del brazo,
La warfarina se puede utilizar en la enfermedad aterotrombóti- déficit sensitivo cortical de la pierna, reflejo de prensión y de
ca cuando no hay opción a la cirugía o cuando existen embo- succión, incontinencia urinaria, apraxia de la marcha.
lias de origen cardiaco.
Hemisferio cerebral, cara medial
Agentes antiplaquetarios (a. cerebral posterior)

Pueden reducir el riesgo de TIA posteriores y de ictus en pa- Hemianopsia homónima, ceguera cortical, déficit de memo-
cientes sintomáticos. ria, déficit sensitivo profundo, con disestesias dolorosas es-
pontáneas, coreoatetosis.
Endarterectomía quirúrgica
Tronco cerebral, mesencéfalo (a. cerebral posterior)
En estenosis >70% de carótidas sintomáticas.
Parálisis del III par y hemiplejía contralateral; parálisis/pa-
Infarto lacunar resia de la mirada vertical; nistagmo de convergencia, deso-
rientación.
Manejo agresivo de la hipertensión.
Tronco del encéfalo, unión pontobulbar (a. basilar)
Hemorragia intracerebral
Parálisis facial, paresia de la abducción ocular, parálisis de
Corrección agresiva de cualquier coagulopatía; evaluación la mirada conjugada, déficit sensitivo hemifacial, sd. de
neuroquirúrgica para posible evacuación del hematoma, espe- Horner, disminución de la sensibilidad termoalgésica en medio
cialmente en casos de hemorragia cerebelosa. cuerpo, ataxia.
MANEJO DE LA TENSION ARTERIAL EN EL Tronco del encéfalo, parte lateral del bulbo
ACCIDENTE CEREBROVASCULAR AGUDO (a, Vertebral)
Se debe mantener una presión de perfusión cerebral ade- Vértigo, nistagmo, sd. de Horner, ataxia, caída hacia el lado
cuada. Evitar hipotensión (TA sistólica <140 mmHg) y disminuir de la lesión, afectación de la sensibilidad termoalgésica en
de manera cautelosa una TA sistólica >210. medio cuerpo con o sin afectación de la cara.

1
*La semiología reflejada aparecerá en función de las áreas lesio-
nadas y no tiene por qué aparecer todo el cuadro clínico en los infartos
de los territorios de las diferentes arterias.

1642
Capítulo V

ENFERMEDAD
DE PARKINSON Y OTRAS
ALTERACIONES
DEL MOVIMIENT O

Indice
Anatomía y fisiología de los ganglios basales Parálisis supranuclear progresiva
Enfermedad de Parkinson Enfermedad de Wilson
Atrofia multisistémica Enfermedad de Huntington

Dra. MARIA ROSARIO ANTON

ANATOMIA Y FISIOLOGIA DE ENFERMEDAD DE PARKINSON


LOS GANGLIOS BASALES
Enfermedad crónica, degenerativa y progresiva del SNC.
Los ganglios basales son estructuras imprescindibles para el Prevalencia: 1% en mayores de 60 años. En el 10% comien-
control motor normal. Son un grupo de núcleos situados en za en <40 años.
diencéfalo y mesencéfalo: estriado (caudado, putamen, accum-
bens y tubérculo olfatorio), pálido interno y externo, núcleo Etiología
subtalámico de Luys y sustancia nigra (pars compacta y pars
Desconocida. Acumulación familiar del 1-2% y la concor-
reticular).
dancia entre gemelos homocigóticos es escasa.
Los neurotransmisores implicados en las aferencias y efe-
rencias entre estos núcleos son: glutámico, GABA y dopamina.
Anatomía patológica
La principal vía dopaminérgica es la nigroestriada, la dopa-
mina es excitadora sobre los receptores estriatales D1 en la Se produce primariamente una degeneración de las neuro-
vía directa, e inhibidora sobre los D2, origen de la vía indirecta. nas de la sustancia negra (pars compacta) con gliosis reactiva
La dopamina reforzaría la actividad cortical iniciada. en locus coeruleus, pero sobre todo en sustancia nigra, junto

1643
ENFERMEDAD DE PARKINSON Y OTRAS ALTERACIONES DEL MOVIMIENTO

con aparición de cuerpos de Lewy en estos núcleos y en el cór- — La levodopa es el tratamiento más eficaz (un 10% no
tex cerebral (inclusiones citoplasmáticas eosinófilas). responden). Debe administrarse junto a un inhibidor de
En la intoxicación por MPTP (metilfeniltetrahidropiridina) se la dopa-decarboxilasa periférica (carbidopa o bensera-
destruyen selectivamente las neuronas dopaminérgicas de la zida), ya que el 90% se decarboxila en sangre periférica
sustancia negra originándose un síndrome parkinsoniano idén- a dopamina con graves efectos secundarios. La dosis
tico al idiopático pero sin cuerpos de Lewy ni pérdida neuronal es variable: comenzar con dosis bajas (50 mg./día) e ir
en locus coeruleus. aumentando. Efectos secundarios son náuseas y vómi-
tos, que se tratan con Domperidona (antiemético anti-
Cuadro clínico dopaminérgico que no atraviesa la barrera hematoen-
Debuta insidiosamente, siendo el síntoma de inicio más fre- cefálica), hipotensión ortostática, arritmias y alteracio-
cuente el temblor de reposo (presente en un 70%) y con menor nes psiquiátricas (en éstas puede usarse la Clozapina,
frecuencia lentitud de movimientos, torpeza motora, dolor arti- neuroléptico no parkinsonizante). Tras 3-5 años de tra-
cular, depresión, fatiga... Al inicio suele ser unilateral, con el tamiento aparecen fluctuaciones de la motilidad (perío-
tiempo se generaliza. dos de bloqueo motor con o sin distonía, por hipofun-
Tétrada clásica: temblor de reposo, rigidez, acinesia y alte- ción dopaminérgica) y discinesias, sobre todo corea
ración postural. (por hiperfunción). Ambos casos pueden coexistir. Hay
En ocasiones puede asociarse temblor postural. que recurrir a la fragmentación de la dosis o a la adi-
La rigidez es en tubo de plomo o en rueda dentada. ción de nuevos fármacos (selegilina, agonistas..)
El síntoma más invalidante es la acinesia (lentitud o pobreza — Los agonistas dopaminérgicos (Bromocriptina, Lisuri-
de movimientos) responsable de la hipomimia facial, marcha a de, Pergolide) estimulan los receptores dopaminérgi-
pasos cortos sin braceo, hipofonía, micrografia. cos estriatales, son de menor eficacia terapéutica que
Tienden a mantener una postura en flexión del cuello y tron- la levodopa y tienen menos efectos secundarios. Pue-
co con flexión de miembros, y en fases avanzadas aparece den emplearse en fases precoces para retrasar el ini-
inestabilidad postural, lo que facilita las caídas, pudiendo apa- cio de tratamiento con levodopa o como adyuvantes
recer también trastornos autonómicos (impotencia, hipotensión de levodopa en fases avanzadas con fluctuaciones.
ortostática) y demencia (33%). — Cuando el temblor de reposo es síntoma predominan-
te pueden usarse anticolinérgicos.
Diagnóstico — Cuando el temblor de acción o un temblor esencial son
incapacitantes se usa el propranolol o primidona.
Es clínico. Las pruebas de imagen son normales.
ATROFIA MULTISISTEMICA
Diagnóstico diferencial
Con el parkinsonismo secundario: Grupo heterogéneo de enfermedades que tienen en común
la lesión degenerativa de núcleos pontinos, cerebelo, olivas in-
— Infecciones: parkinsonismo postencefalítico. feriores, núcleos pigmentados del mesencéfalo y protuberan-
— Tratamiento prolongado con neurolépticos, es la cau- cia, ganglios basales y médula. Se distinguen tres entidades:
sa más frecuente de parkinsonismo secundario, el ini-
cio suele ser bilateral y simétrico, de curso subagudo, Atrofia olivopontocerebelosa (AOPC)
con temblor mandibular, discinesias bucolinguales y
asociar depresión y acatisia. Agrupa entidades con herencia autosómica dominante o de
— Intoxicación por MPTP. aparición esporádica.
— Arterioesclerótico, por infartos bilaterales múltiples Aparecen en el adulto, son de lenta evolución, siendo el sínto-
en ganglios basales. ma principal la ataxia cerebelosa, pudiendo añadirse parkinsonis-
— Demencia pugilística: parkinsonismo, demencia, disartria. mo y demencia; también hay signos de alteración de neurona mo-
— Degeneración nigroestriatal. tora de esfínteres, disartria, disfagia, alteraciones oculomotoras..
— Hidrocefalia. Existe un grupo de familias en que existe un déficit de des-
hidrogenasa del ácido glutámico.
Tratamiento En TAC o RM puede existir atrofia de las estructuras de la
fosa posterior.
Sintomático.
El tratamiento es sintomático, no suelen responder a levo-
dopa.
— Selegilina o Deprenilo, es un inhibidor de la MAO-B,
enzima que cataboliza la levodopa y que convierte el
Síndrome de Shy-Drager
MPTP en MPP+, el tóxico per se. La selegilina retarda
el curso natural de la enfermedad, debe administrarse Degeneración neuronal y gliosis en columna intermediolate-
desde las etapas más precoces. ral de la médula torácica, también en ganglios basales y nú-

1644
NEUROLOGIA

cleos del tronco, a veces en cerebelo. Además existe pérdida


25
celular en ganglios autonómicos periféricos. 26
Comienza en la sexta-séptima década, con más frecuencia La tétrada clásica de la enfermedad de Parkinson es:
en varones.
La disautonomía se manifiesta por disfunción vesical (chorro 1. Rigidez, hipercinesia, atrofia y fasciculaciones.
2. Corea, demencia, alteraciones psiquiátricas y acinesia.
intermitente e incontinencia), impotencia, hipotensión postural
3. Rigidez, temblor, acinesia y alteración postural.
y anhidrosis. A veces asimetría pupilar, síndrome parcial de 4. Disautonomía, neuropatía, ceguera y temblor.
Horner, parkinsonismo con predominio de rigidez y bradicinesia 5. Paraparesia, cefalea, sordera y neuropatía.
y con pobre respuesta a la levodopa. Puede existir ataxia cere-
belosa, parálisis laríngea y apnea del sueño.
El tratamiento de la hipotensión postural es sintomático:
medias elásticas, dormir en posición de antitrendelenburg, in- 27
gesta abundante de ClNa más fludrocortisona, en casos graves
fármacos adrenérgicos como efedrina, levodopa o anfetamina La parálisis supranuclear progresiva (señale la falsa):
o agonistas alfa centrales como la clonidina. 1. Es una enfermedad degenerativa.
Hay que realizar diagnóstico diferencial con la enfermedad 2. Puede presentar distonía axial.
por cuerpos de Lewy difusos, y se hará por anatomía patológi- 3. Tiene alteración de la mirada supranuclear.
ca, encontrándose dichos cuerpos en la columna intermediola- 4. Nunca se caen los pacientes.
5. No responden al test de la apomorfina.
teral medular.

Degeneración nigroestriatal
Aparece al final de la vida adulta. Existe degeneración de la 28
sustancia nigra y del estriado. En RM atrofia de caudado y pu- La disminución de los niveles de ceruloplasmina sérica es típica de:
tamen. El síntoma fundamental es un parkinsonismo que no
responde a levodopa y que puede asociar ataxia progresiva o 1. Enfermedad de Wilson.
2. Enfermedad de Huntington.
hipotensión ortostática. 3. Enfermedad de Parkinson.
4. Atrofia olivopontocerebelosa.
PARALISIS SUPRANUCLEAR PROGRESIVA 5. Degeneración estriatonígrica.
(Enfermedad de Steele-Richardson-Olszewski)

Enfermedad degenerativa de etiología desconocida.


Prevalencia: 1.4/100.000 29
Pérdida neuronal y gliosis en techo y tegmento mesencefáli- ¿Cuál de estos fármacos no se utiliza en la terapia de la enfermedad de
co, núcleo subtalámico, núcleos vestibulares y algunos oculares. Parkinson?:
Microscópicamente: ovillos neurofibrilares rectos (no heli-
coidales como en el Alzheimer). 1. Selegilina.
2. L-DOPA.
Síndrome rígido-acinético que de forma precoz asocia ines- 3. Bromocriptina.
tabilidad con caídas frecuentes, hipofonía, disartria y alteracio- 4. Pergolide.
nes mentales (labilidad emocional, depresión, apatía...). 5. Haloperidol.
Al final de la evolución: síndrome rígido-acinético, síndrome
frontal con demencia, distonía axial, parálisis pseudobulbar,
inestabilidad postural con caídas hacia atrás y oftalmoplejía
supranuclear. 30
El diagnóstico de certeza es patológico. En la RM, atrofia
mesencefálica con aumento de las cisternas perimesencefáli- ¿Cuál de estas entidades cursa predominantemente con una disfunción
autonómica?:
cas. No responden al test de la apomorfina, lo que los diferen-
cia de la enfermedad de Parkinson. Responden poco o no res- 1. Enfermedad de Parkinson.
ponden a la medicación antiparkinsoniana. 2. Síndrome de Shy-Drager.
3. Atrofia olivopontocerebelosa.
4. Degeneración estriatonígrica.
ENFERMEDAD DE WILSON
5. Parálisis supranuclear progresiva.
Presenta un patrón de herencia autosómico recesivo, con
gen anómalo en el brazo corto del cromosoma 13. RESPUESTAS: 26: 3; 27: 4; 28: 1; 29: 5; 30: 2.
Prevalencia: 3/10.000.

1645
ENFERMEDAD DE PARKINSON Y OTRAS ALTERACIONES DEL MOVIMIENTO

Etiología y anatomía patológica D-penicilamina más piridoxina, hasta un 40 % empeoran en


el primer mes, comenzando la mejoría a partir del tercer mes.
Se produce un exceso de cobre (por fallo en la excreción bi-
Efectos secundarios: precoces como intolerancia, depresión
liar con una absorción intestinal normal) que se acumula en hí-
medular y tardíos como nefrotoxicidad, LES, trombocitopenia,
gado, SNC, córnea, riñón y sistema musculoesquelético.
miastenia, Good-Pasture...
En los ganglios basales se observa pérdida neuronal, dege-
Si los efectos secundarios obligan a retirar el fármaco se
neración axonal y astrocitosis con depósitos de cobre pericapi-
puede usar acetato de cinc, trientine... en último caso, tras-
lares y en los astrocitos.
plante hepático.
Cuadro clínico ENFERMEDAD DE HUNTINGTON
Comienza en las primeras décadas de la vida y un 50% de-
buta con hepatopatía en forma de hepatitis aguda, fulminante Herencia autosómica dominante con gen anómalo en brazo
o crónica activa. Un 15% debuta con síntomas psiquiátricos corto de cromosoma 4.
(ansiedad, histeria, depresión o psicosis franca).
Un 40% debuta con síntomas neurológicos: temblor, disto- Anatomía patológica
nía, corea, disartria, parkinsonismo, alteraciones de la marcha, Pérdida neuronal y gliosis en caudado y en putamen.
crisis convulsivas, síndrome seudobulbar y deterioro cognitivo.
No es frecuente los signos de alteración de la vía piramidal, Cuadro clínico
oculomotores, sensitivos o esfinterianos. En un 5% debutan Curso progresivo y mortal en 10-30 años.
con trastornos oculares (anillo de Kayser-Fleischer, catarata..), La tríada sintomática clásica es corea, demencia y alteracio-
renales, esqueléticos o hematológicos (anemia hemolítica). nes psiquiátricas, pudiendo debutar con cualquiera de estas
manifestaciones.
Diagnóstico Los movimientos coreicos son involuntarios, fluidos, impre-
— Ceruloplasmina sérica<20 mg./dl. decibles, se acompañan de impersistencia motora y es cons-
Falsos positivos: heterocigotos, hipoproteinemias se- tante la dificultad para los movimientos oculares voluntarios.
veras, hepatopatías graves, enfermedad de Menkes.
Falsos negativos: 5% en caso de embarazo e ingesta Diagnóstico
de anticonceptivos orales. Clínico y por antecedentes familiares. Diagnóstico de certe-
— Anillo de Kayser-Flescher corneal, por lámpara de za con estudio genético.
hendidura.
— Aumento en la excreción de cobre urinario. Tratamiento
— Biopsia hepática con aumento de la concentración de
No existe tratamiento eficaz.
cobre>250 microgr./gr. de tejido seco.
Para el corea: neurolépticos como haloperidol o pimozide;
— Estudio genético.
deplecionantes de dopamina a nivel presináptico como la te-
trabenazina. Paradójicamente se han obtenido buenos resulta-
Tratamiento
dos con agonistas dopaminérgicos a dosis bajas.
Disminuir la ingesta de alimentos ricos en cobre (hígado, En casos de depresión, la fluoxetina.
chocolate, marisco). Es fundamental el consejo genético.

1646
Capítulo VI

INFECCIONES
DEL SISTEMA NERVIOSO
CENTRAL
Indice
Infecciones no víricas Infecciones víricas

Dra. MARIA CASTRO DEL RIO

INFECCIONES NO VIRICAS Incidencia según la edad:


Infecciones bacterianas — Neonatos (<1 mes).
Meningitis bacteriana
• Bacilos Gramnegativos (E. coli) y Streptococcus
Concepto del grupo B son los más frecuentes.
Se trata de la infección de Piamadre y Aracnoides y del lí- • Con una incidencia menor, seguiría la Lysteria.
quido del espacio subaracnoideo y del sistema ventricular. — Niños (2 meses a 14 años):
Etiología • Meningococo.
• Haemophilus influenzae (1.° en EEUU).
Los tres gérmenes más frecuente (76% del total) son • Neumococo.
Meningococo, Neumococo y Haemophilus influenzae B que, en
España, representan el 54%, 15% y 7% respectivamente, de — Adultos:
ese 76%. • Neumococo.
El 24% restante incluye a Listeria monocytogenes (4.a causa • Meningococo.
más frecuente de M. no traumáticas o postquirúrgicas),
Staphylococcus aureus y epidermidis, Streptococcus A, E. coli, — Ancianos:
Streptococcus B, Klebsiella, Proteus y Pseudomonas.
• Neumococo.
Epidemiología • Bacilos Gramnegativos.
• Lysteria.
Se dan 4.6-10 casos cada 100.000 habitantes al año. Es más
frecuente en varones y en otoño, invierno y primavera. Otros datos epidemiológicos:

1647
INFECCIONES DEL SISTEMA NERVIOSO CENTRAL

— Traumatismos craneoencefálicos: como en pacientes alcohólicos, debilitados, ane-


mia de células falciformes, fractura de base de
• Neumococo. cráneo y pacientes esplenectomizados.
• Bacilos Gramnegativos. • La M. por Haemophilus suele aparecer después
• Haemophilus influenzae. de infecciones de vías respiratorias.
— Procedimientos neuroquirúrgicos: — Manifestaciones clínicas atípicas:
• Bacilos Gramnegativos. • Los signos de irritación meníngea suelen estar
• Pseudomonas aeruginosa. ausentes en pacientes ancianos o en los que su-
• Staphylococcus aureus. fren estupor o coma.
— Infecciones de shunts de LCR: • Los signos neurológicos focales no ocurren a me-
nos que la meningitis se complique con vasculitis
• Staphylococcus coagulasa-negativos. cerebral, trombosis de senos, absceso cerebral o
• Staphylococcus aureus. subdural y encefalitis; la afectación cerebral focal
— Trasplantes renales: es menos común en la M. meningocócica que en
• Listeria monocytogenes. M. por haemophilus o neumococo.
• Las crisis convulsivas son inusuales en adultos,
— Huéspedes inmunodeprimidos: siendo más frecuentes en M. por Haemophilus y
• Defectos de inmunidad celular: en neumocócicas.
• Las anomalías de pares craneales se observan so-
* Listeria monocytogenes. bre todo en M. neumocócicas y se deben a la in-
• Defectos de inmunidad humoral: vasión de los nervios por exudado purulento.
• La más importante de las formas inusuales de las
* Neumococo.
infecciones meningocócicas es la septicemia me-
* Haemophilus influenzae.
ningocócica fulminante (s. Waterhouse-Friderich-
* Meningococo.
sen); el paciente suele fallecer en 48 h. o menos.
• Neutropenia: • Finalmente, ante meningitis bacterianas recurrentes,
* Pseudomonas aeruginosa. se descartarán alteraciones de la inmunidad, una
* Enterobacteriáceas. causa local, síndrome de Behçet o M. de Mollaret.

Exámenes complementarios
Patogenia
— Punción lumbar: Es la prueba más importante. Los ha-
Los tres gérmenes más frecuentes forman parte de la flora
llazgos son:
normal de la nasofaringe. Entre los factores que predisponen
a la invasión de la corriente sanguínea podrían estar las infec- • Presión: se eleva tanto (>180 mm. de agua) que
ciones víricas previas que, además, perturbarían la BHE, o una presión normal o baja en la PL inicial debería
bien, traumatismos previos, enfermedades del oído medio y hacer pensar en posibilidad de oclusión parcial de
senos paranasales, sitios de craneotomía o abscesos que rom- la aguja o bloqueo del ESA espinal. Una presión
pen al espacio subaracnoideo... >400 mm. de agua sugiere inflamación del cere-
bro y posible herniación cerebelosa (la presión
Clínica normal es de 65 a 195 mm de agua).
Las primeras manifestaciones clínicas son: fiebre (>90%), ce- • Células: Pleocitosis (aumento del número de célu-
falea (80-90%), rigidez de nuca, signos de Kernig y Brudzinski. las, habitualmente entre 1.000 y 10.000). Predo-
Además pueden existir convulsiones y trastornos del nivel minan los leucocitos polimorfonucleares (85-95%
de conciencia. del total); pero al avanzar la infección, aumenta la
proporción de mononucleares.
— Manifestaciones clínicas peculiares: • Proteínas: Hiperproteinorraquia en el 90% de los
casos. La mayoría presentan una cifra entre 100 y
• Hay que pensar en M. meningocócica ante la 500 mg./dl. (la cifra normal de proteínas en el LCR
erupción de petequias, púrpura, equimosis exten- es de 15 a 45 mg./dl.).
sa y livideces en la piel. El 50% de dichas menin- • Glucosa: Hipoglucorraquia: la cifra suele ser
gitis se acompañan de exantema petequial (el <40% de la glucemia si ésta es <250 mg./dl. (la
rash es más evidente en superficies extensoras). cifra normal es de 45 a 80 mg./dl.).
• Se sospechará M. neumocócica ante el antece- La glucorraquia se ha de valorar siempre en rela-
dente de infección de pulmón, senos u oídos; así ción con la cifra de glucosa en sangre; hay que de-

1648
NEUROLOGIA

terminar la glucemia 2 horas antes de realizar la PL.


25
El aumento de la glucosa en LCR por encima del va- 31
lor normal no tiene significación diagnóstica. ¿Cuál es el ganglio que se ha implicado en el síndrome de Ramsay
Probablemente la disminución de la glucosa en Hunt?:
LCR se deba a un mecanismo de inhibición de la
entrada de glucosa al LCR debido a un deterioro 1. Ganglio de Gasser.
2. Ganglio ciliar.
del sistema de transferencia de membrana. 3. Ganglio de Ramsay Hunt.
• LDH: La actividad total de esta enzima está aumen- 4. Ganglio geniculado.
tada en las meningitis bacterianas (a expensas de 5. Ninguna es correcta.
las fracciones 4 y 5 que derivan de los granulocitos).
• Acido láctico: está aumentado en meningitis bacte-
rianas y fúngicas, no así en víricas. LDH, Acido lác- 32
tico y Proteína C reactiva son útiles sobre todo para ¿Cuál de las siguientes afirmaciones es incorrecta sobre el tratamiento
el diagnóstico de meningitis parcialmente tratadas. con Acyclovir del herpes zoster?:
• Tinción Gram del sedimento: permite identificar el
agente causal en muchos casos. 1. Acorta la duración del dolor agudo.
2. Disminuye la incidencia de neuralgia postherpética.
• Test de detección de antígenos: LA (test látex aglu- 3. Acelera la curación de las vesículas.
tinación) tiene una sensibilidad del 81-100% para 4. Se puede asociar a corticoides si no existe neoplasia u otro
detectar ag. de Haemophilus influenzae B en LCR; trastorno que comprometa la inmunidad.
50-69% para neumococo y 30-70% para N. menin- 5. Todas son correctas.
gitidis (sólo A y C). Suelen ser rápidos y baratos.
• Cultivo de LCR: es positivo en el 70-90% de M.
bacterianas. 33
Indicaciones y contraindicaciones de la PL: La PL de- ¿Cuál de las siguientes enfermedades cursa típicamente con un LCR nor-
bería realizarse siempre ante sospecha de meningitis mal?:
y sólo debería diferirse: Cuando hay que administrar
1. Encefalitis herpética.
de forma inmediata el tratamiento antibiótico, pues- 2. Meningitis por citomegalovirus.
to que su retraso supone un gran riesgo (shock sépti- 3. Enfermedad de Creutzfeldt-Jakob.
co o alto riesgo para el desarrollo del mismo). 4. Neurosífilis tabética.
Ante posible riesgo de herniación: por rápido dete- 5. Síndrome de Vogt-Koyanagi-Harada.
rioro del nivel de conciencia, presencia de signos
neurológicos focales, convulsiones o papiledema.
34
— Hemocultivos: Se deben hacer siempre. Son positivos ¿Cuál de los siguientes gérmenes, causantes de meningitis, produce con
en el 40-60% de los pacientes con meningitis por Hae- mayor frecuencia defectos neurológicos residuales?:
mophilus, Meningococo y Neumococo.
1. Meningococo.
— Hemograma y bioquímica elementales.
2. Echovirus.
— Métodos radiológicos: 3. Haemophilus influenzae.
4. Virus coxsackie.
• Rx de tórax (para descartar área de neumonitis o 5. Neumococo.
absceso).
• Rx de senos y cráneo para investigar osteomielitis
craneal, sinusitis y mastoiditis. 35
• CT craneal: se hará ante sospecha de HTIC y/o
Ante un paciente con un síndrome meníngeo en el que el LCR contiene:
absceso o empiema subdural.
células: 400 (mononucleares), proteínas: 236 mg/dl y glucosa: 20
mg/dl (glucemia: 102), ¿cuál es la etiología menos probable?:
Diagnóstico diferencial
1. Tuberculosis.
Meningitis no bacterianas, M. químicas, Hemorragia suba- 2. Criptococosis.
racnoidea espontánea, M. recurrentes, Absceso cerebral y em- 3. Meningitis bacteriana decapitada.
piema subdural. 4. Enterovirus.
5. Meningitis carcinomatosa.
Tratamiento
— Empírico: En niños o adultos sin factores de riesgo: RESPUESTAS: 31: 4; 32: 2; 33: 3; 34: 5; 35: 4.
Cefotaxima i.v: 8-12 gr./día repartidos en 4-6 dosis.

1649
INFECCIONES DEL SISTEMA NERVIOSO CENTRAL

Este antibiótico no es activo frente a Listeria; por lo Si no se trata, se produce un desenlace fatal a las 4-8 sema-
que en pacientes con factores de riesgo, se añadirá nas de su aparición.
Ampicilina: 2 gr./4 h. i.v. En cuanto a las pruebas complementarias:
— Específico: — La PL revela pleocitosis moderada (50-500 células) con
• Meningococo: Penicilina i.v. (en alérgicos: Cloran- predominio de linfocitos, aunque al principio pueden pre-
fenicol). dominar los PMN, hiperproteinorraquia (100-200 mg./dl.);
• Neumococo: La misma pauta si es sensible; pero e hipoglucorraquia (la glucosa desciende lentamente y no
en España hay una alta tasa de resistencias, y, en suele alcanzar valores tan bajos como en la piógena).
ese caso, se empleará Cefotaxima. — Suele asociarse disminución de sodio y cloro séricos y
• Haemophilus influenzae: Cefotaxima o Ceftriaxona. cloro en LCR, habitualmente por SIADH.
• Listeria monocytogenes: Ampicilina + Aminoglu- — La tinción Ziehl-Neelsen revela BAAR y la muestra de
cósido. LCR se ha de cultivar en medio Löwenstein para obte-
• Staphylococcus aureus: si es meticilinsensible: ner el diagnóstico definitivo.
Oxacilina: 10-12 gr./día. Si es meticilínresistente: — El Mantoux no siempre es positivo.
Vancomicina: 1 gr./12 h i.v. — El CT craneal suele mostrar hidrocefalia y realce me-
• Gramnegativos resistentes a Ampicilina: Cefalos- níngeo.
porinas de 3.a generación. — El diagnóstico diferencial se hará con encefalitis, me-
ningitis bacteriana, fúngica y maligna, sarcoidosis,
La duración del tratamiento debe de ser de 10 a 14 días. No absceso cerebral y empiema subdural. Ante la sospe-
son necesarias PL repetidas para determinar el efecto del trata- cha, está indicado comenzar el tratamiento.
miento. El nivel de glucosa puede permanecer bajo durante va-
Tuberculoma
rios días después de desaparecer los demás signos de infección.
El uso de esteroides está muy controvertido. Es una masa de tejido de granulación tuberculosa en parén-
Los contactos íntimos y los miembros de la familia que hayan es- quima cerebral, que se comporta como una lesión expansiva.
tado en contacto con el enfermo de M. meningocócica deben recibir En el LCR hay aumento de células y proteínas.
profilaxis con Rifampicina: 600 mg./12 h v.o. durante 2 días. Tratamiento:Isoniazida 5 mg./kg./día (se asociará piridoxina:
50mg./d. para evitar el desarrollo de Neuropatía)+Rifampicina
Encefalitis bacteriana 600 m.g/día+Etambutol 15mg./kg./día y/o Pirazinamida 30
Destacan tres de ellas: mg./kg./día. El tratamiento dura 18-24 meses (a los dos meses
se puede retirar uno de los dos últimos).
— Enfermedad del Legionario. Los corticoides sólo se utilizarán si la vida del paciente se
— Infecciones por Mycoplasma. ve amenazada y sólo con los otros fármacos.
— Meningoencefalitis por Listeria monocytogenes: se ob-
serva en individuos debilitados e inmunodeprimidos. Neurosífilis
Hay aumento de mononucleares y proteínas en LCR. Neurosífilis asintomática
Tromboflebitis intracraneal No hay síntomas o signos. El diagnóstico se basa en el exa-
men del LCR.
Infección de senos durales. Los senos que se suelen afectar
son los más grandes: s. lateral (transverso), cavernoso y longi- Sífilis meníngea
tudinal. Los gérmenes más frecuentes son estreptococos y es-
tafilococos Suele existir un síndrome meníngeo sin fiebre y con LCR
anormal.
Tuberculosis
Sífilis meningovascular
Meningitis tuberculosa
Suele aparecer a los 6-7 años de la infección original (rango:
Es una enfermedad seria, más indolente y subaguda en su 6 meses-12 años). Se manifiesta habitualmente con accidentes
evolución que la meningitis bacteriana piógena. cerebrovasculares (hemiplejía, afasia, pérdida de sensibili-
Las primeras manifestaciones son cefalea, letargia, confu- dad...). El LCR suele ser anormal (aumento de células y proteí-
sión, fiebre (de bajo grado), rigidez de nuca (75%) y s. de
nas y una prueba serológica positiva). La lesión vascular típica
Kernig y Brudzinski. Debido a su cronicidad inherente, en el
momento de su admisión en el hospital, los pacientes suelen se denomina arteritis de Heubner.
presentar afectación de pares craneales (sobre todo parálisis Neurosífilis tabética (tabes dorsal)
ocular), hidrocefalia, déficit focales, crisis comiciales y coma.
En la mayoría de los pacientes se encuentra tuberculosis ac- Se desarrolla entre 15 y 20 años después del comienzo de la
tiva en alguna otra parte; en otros, sólo se detectan lesiones infección. Los síntomas y signos son: dolores fulgurantes en
inactivas en los pulmones. las piernas, ataxia por defecto de sensibilidad profunda e in-

1650
NEUROLOGIA

continencia urinaria por rebosamiento; Las pupilas son anor-


25
males en el 90%, habitualmente de tipo Argyll-Robertson. En 36
un 1-10% de casos aparecen articulaciones de Charcot. En un ¿Cuál de las siguientes complicaciones neurológicas se ha asociado con
5-10% el LCR es normal. Neumonía por Mycoplasma?:
1. Mielitis transversa.
Neurosífilis parética 2. Meningitis aséptica.
Se desarrolla entre 15 y 20 años después del comienzo. Se 3. Miositis aguda.
caracteriza por un síndrome demencial en el que el deterioro 4. Encefalomielitis.
5. Todas son correctas.
mental es paralelo a un deterioro físico que conduce a un esta-
do de postración en la fase paralítica.
Anomalías en el LCR en la Neurosífilis: 37
La abolición del reflejo fotomotor con preservación del reflejo de acomo-
— Aumento de células (200-300) de predominio linfocitario. dación se denomina:
— Aumento de proteínas (40-200 mg./dl).
— Aumento de gammaglobulinas. 1. Pupila de Argyll-Robertson.
2. Pupila de Adie.
— Glucosa normal.
3. Pupila de Marcus-Gunn.
— Pruebas serológicas positivas: Las pruebas reagínicas 4. Pupila tónica.
(VDRL) son las más comunes y, si son positivas, son diag- 5. Ninguna de las anteriores.
nósticas de Neurosífilis. En caso negativo, se realizarán
pruebas específicas treponémicas (FTA-ABS y TPI) que
son positivas, en casi todos los casos de Neurosífilis. 38
¿Qué es incorrecto sobre la Mucormicosis cerebral?:
Tratamiento: Penicilina G cristalina: 3-4 millones de unida-
1. Es una complicación rara de la acidosis diabética.
des/4 h. durante 14 días. 2. La infección suele iniciarse en los cornetes nasales.
Tratamiento alternativo: Eritromicina o Tetraciclinas. 3. El pronóstico suele ser bueno.
El paciente debe ser revisado cada 3 meses y el LCR, exami- 4. El tratamiento antifúngico es Anfotericina B.
nado cada 6 meses hasta que clínica y LCR sean negativos. 5. La corrección de la hiperglucemia y la acidosis favorece la recu-
peración.
Miscelánea
Enfermedad de Lyme 39
— Parálisis de pares craneales (sobre todo VII par). El aislamiento de cocos en racimos grampositivos en el LCR de un pa-
— Polirradiculopatía. ciente con meningitis ocurrirá con más probabilidad en el siguiente
— Mononeuritis múltiple. caso:
— Meningitis subaguda o crónica. 1. Neutropenia.
— Mielitis transversa. 2. Inmunodepresión.
3. Válvula de derivación de LCR.
Infecciones micóticas 4. Traumatismo craneoencefálico.
5. Es el germen más común en el paciente previamente sa-
Pueden dar lugar a diferentes síndromes clínicos. La meningi- no.
tis micótica se desarrolla de forma insidiosa, como la meningitis
tuberculosa, y los síntomas y signos son también los mismos.
El LCR muestra presión elevada, pleocitosis moderada con 40
predominio de linfocitos; la glucosa suele ser baja y el nivel de
¿Cuál de los siguientes métodos consigue inactivar el germen transmisi-
proteínas, alto. ble responsable de la enfermedad de Creutzfeldt-Jakob?:
Los más habituales son: Criptococosis, Mucormicosis y
Candidiasis. 1. Alcohol.
2. Hipocloritos.
3. Ebullición.
Infecciones por protozoos 4. Radiación ultravioleta.
Destacan la meningoencefalitis amebiana (causada por 5. Formalina.
amebas del género Naegleria; su diagnóstico depende de una
historia de baños en agua dulce y caliente y el hallazgo de tro- RESPUESTAS: 36: 5; 37: 1; 38: 3; 39: 3; 40: 2.
fozoitos en LCR) y la Toxoplasmosis.

1651
INFECCIONES DEL SISTEMA NERVIOSO CENTRAL

INFECCIONES VIRICAS van asociadas a la encefalitis). El virus tipo II puede causar en-
cefalitis, sobre todo en el neonato.
Infecciones víricas agudas El sustrato lesional consiste en necrosis hemorrágica inten-
Síndromes clínicos sa en una localización típica: parte inferointerna de lóbulos
temporales y parte orbitaria de lóbulos frontales.
Meningitis aséptica Clínicamente se caracteriza por cefalea (75%), fiebre (90%),
Cefalea, fiebre y otros signos de irritación meníngea, con un alteración del nivel de conciencia (97%), vómitos, crisis comi-
comienzo agudo y una evolución habitualmente benigna. ciales (45%) y otros signos de focalidad neurológica tales co-
El LCR muestra pleocitosis linfocítica (puede haber predomi- mo hemiparesia o afasia (48%).
nio de PMN el primer día) y glucosa normal; éste es un dato im- El LCR revela aumento de presión, pleocitosis (10-500) de predo-
portante porque una glucosa baja con pleocitosis linfocítica sue- minio linfocítico, aunque a veces hay un número significativo de
le corresponder a una meningitis tuberculosa o fúngica o a un neutrófilos. Puede haber un número importante de eritrocitos y xan-
trastorno no infeccioso; por ello debe repetirse la determinación tocromía, que refleja la naturaleza hemorrágica de la lesión. El nú-
de la glucosa a intervalos, hasta que se haya llegado al diagnós- mero de proteínas suele estar alto y la cifra de glucosa habitual-
tico o el paciente haya entrado en fase de convalecencia. mente es normal. Es raro aislar el virus del LCR. El EEG puede reve-
Su incidencia es de 11 a 27 casos por 100.000 anualmente. lar ondas agudas de alto voltaje y complejos de ondas lentas perió-
Causas: la mayoría son víricas. dicas, estereotipadas y repetitivas que tienen su origen en regiones
frontotemporales; estos complejos se suelen ver entre el 2.° y 15.°
— Los virus más comunes son los enterovirus (Echovirus, día de enfermedad; no son patognomónicos pero sí son sugestivos.
Poliovirus y v. Coxsackie). La anomalía más precoz en la TC es un discreto efecto de ma-
— V. de la Parotiditis. sa y, entre el 3.° y 6.° día, se pueden ver lesiones de baja densi-
— Herpes simple tipo 2. dad, asimétricas; en las regiones descritas de los lóbulos tempo-
— Coriomeningitis linfocítica. rales y frontales dichas lesiones realzan con contraste en un 50%
— Adenovirus. de casos. La RMN es muy sensible para detectar cambios preco-
ces; se suelen observar áreas de aumento de intensidad de señal
Todos ellos junto con Leptospirosis y Borreliosis de Lyme
en T2 en las regiones anteriormente descritas.
representan el 95% de todas las meningitis asépticas de etio-
La biopsia cerebral revela inclusiones cerebrales típicas en
logía conocida.
un 60%. El diagnóstico definitivo se obtiene al aislar el virus
Las formas no víricas incluyen:
del tejido cerebral.
El tratamiento más adecuado es Acyclovir, que se debe adminis-
— Meningitis piógenas mal tratadas. trar por vía i.v, en dosis de 10 mg./kg./8 horas; cada dosis se admi-
— Absceso cerebral. nistrará en una infusión (con un volumen mínimo de 50 ml.) que pa-
— Infecciones específicas (Tuberculosis, Sífilis y Cripto- sará lentamente (en 1 ó 2 horas). La duración del tratamiento será
cocosis). de 14 días (mínimo de 10 días). Está indicado iniciar el tratamiento
— Invasiones neoplásicas (Linfomas, Leucemias y Carci- empíricamente ante la sospecha clínica, licuoral y radiológica.
nomatosis meníngeas). La mortalidad es del 25 % en pacientes comatosos o semi-
— Meningitis inflamatorias recurrentes que incluyen: comatosos de<30 años y de más del 50% en los >30 años.
• S. de Vogt-Koyanagi-Harada (que combina iridoci- Sólo un 10% o menos de los pacientes que sobreviven, quedan
clitis, despigmentación de piel y pelo alrededor sin secuelas neurológicas si el tratamiento se inicia tras la
de los ojos y sordera). aparición del coma. Un 7% de pacientes tienen una recaída en
• Meningitis alérgica. unos días o varios meses después del término del tratamiento.
• Enfermedad de Behçet. Encefalitis letárgica de Von Economo
• Meningitis de Mollaret.
Esta enfermedad apareció por primera vez tras la pandemia
Encefalitis aguda de gripe de la Primera Guerra Mundial. Nunca se identificó el
Es una enfermedad febril aguda con datos de afectación agente causal, pero se cree que se trató de un virus lento.
meníngea, a la que se ha de añadir algún dato de focalidad Es la única forma de encefalitis que se sabe produce un sín-
que sugiera disfunción de cerebro, tronco cerebral o cerebelo. drome extrapiramidal inmediato o retardado, un parkin-
La causa más frecuente son los virus. sonismo que apareció en meses o años (hasta 25) después en
Encefalitis por Herpes simple: Es la forma más común y grave un número elevado de supervivientes.
de Encefalitis aguda. La incidencia anual es de 0.2-0.5/100.000. Infecciones víricas subagudas y crónicas
Se manifiesta de forma esporádica, durante todo el año; en pa-
Leucoencefalopatía multifocal progresiva (LMP)
cientes de todas las edades y con una distribución universal.
Se debe casi siempre al virus del Herpes simple tipo I (que Se caracteriza por lesiones desmielinizantes en hemisferios
es el causante de las lesiones orales, aunque éstas, rara vez cerebrales, tronco cerebral, cerebelo y médula espinal.

1652
NEUROLOGIA

Un papovavirus, el virus JC, ha sido implicado como agente


25
causal. 41
La gran mayoría de casos se ven en pacientes con SIDA y el ¿Cuál de las siguientes afirmaciones es falsa en relación con la meningi-
resto se suelen asociar con enfermedades crónicas neoplásicas. tis bacteriana?:
Típicamente cursa con hemiparesia que evoluciona a tetra-
paresia, defectos del campo visual, ceguera cortical, afasia, 1. La positividad de los cultivos de LCR es de un 40%.
2. Se deben extraer siempre hemocultivos.
demencia, estados confusionales y coma.
3. La meningitis neumocócica suele precederse de infección ótica
El LCR suele ser normal y la CT revela áreas de baja densi- o de senos.
dad que no captan contraste. 4. Suele haber pleocitosis polimorfonuclear en LCR.
La muerte sobreviene en 3-6 meses y no existe tratamiento 5. Todas son correctas.
específico.
42
Encefalopatías espongiformes subagudas ¿Cuál de las siguientes entidades cursa con lesiones focales en la TC cra-
neal que, típicamente, no realzan con contraste y se localizan en
Kuru
sustancia blanca?:
Fue la primera infección lenta documentada en el hombre. 1. Toxoplasmosis cerebral.
La transmisión de la enfermedad se debía a los rituales caní- 2. Linfoma cerebral.
bales. 3. Metástasis cerebral.
La clínica consiste en ataxia cerebelosa progresiva, anoma- 4. Leucoencefalopatía multifocal progresiva.
lías de movimientos extraoculares y debilidad que progresa a 5. Enfermedad de Creutzfeldt-Jakob.
inmovilidad. La muerte se produce en 3-6 meses.
43
E. Creutzfeldt-Jakob ¿Cuál de los siguientes gérmenes, que causan infecciones del sistema
nervioso central, tienen especial predilección por los lóbulos tempo-
Su distribución es universal y la incidencia es de 1 a 2 casos rales?:
por millón de habitantes. El único mecanismo de transmisión
demostrado es yatrogénico: tras trasplante de córnea y tras co- 1. Bacilo tuberculoso.
2. Virus ECHO.
locación de electrodos infectados. También se ha descrito en
3. Meningococo.
pacientes que habían sido tratados con GH de origen humano. 4. Cryptococo.
La enfermedad afecta a corteza cerebral y cerebelosa, con 5. Virus del herpes simple.
desaparición difusa de células nerviosas y proliferación de cé-
lulas gliales. Típicamente aparece en personas de edad avan- 44
zada y la clínica se caracteriza por un síndrome demencial rápi- Ante un paciente con sospecha de meningitis infecciosa, ¿cuándo solici-
damente progresivo que se asocia a ataxia y espasmos miocló- taría TC craneal antes de proceder a la punción lumbar?:
nicos difusos sensibles a todo tipo de estímulos sensitivos y
1. Siempre que no haya papiledema.
también espontáneos, junto con una gran variedad de signos y 2. Si la fiebre es superior a 39° C.
síntomas de focalidad neurológica. La enfermedad es fatal en 3. Si hay un defecto focal tal como un reflejo cutaneoplantar ex-
menos de 1 año. tensor.
La sangre y el LCR son normales. El EEG es característico. 4. Si la sospecha es meningitis vírica.
No existe tratamiento específico. El síndrome de 5. En todos los casos descritos se puede proceder directamente a
Gerstmann-Sträusler es una rara enfermedad de tipo familiar la punción lumbar.
que podría corresponder a una encefalopatía espongiforme su-
45
baguda.
Ante un paciente con un cuadro agudo de cefalea, fiebre, rigidez de nuca y
discretos elementos afásicos en la exploración del lenguaje, que pre-
Panencefalitis esclerosante subaguda (PEES)
senta un LCR con pleocitosis linfocítica, discreta hiperproteinorraquia
Afecta a niños y adolescentes. La historia típica muestra y glucosa normal. La TC craneal es normal, ¿qué tratamiento instau-
una infección por sarampión primaria antes de los dos años, raría de forma empírica?:
seguida de un período asintomático de 6-8 años. 1. Analgésicos e hidratación abundante.
Se produce un deterioro intelectual grave y progresivo+con- 2. Cefotaxima.
vulsiones+mioclonías+ataxia+coriorretinitis progresiva. La 3. Tuberculostáticos.
muerte sobreviene en 1 a 3 años. 4. Acyclovir.
El EEG es característico. Suele elevarse la gammaglobulina 5. Ampicilina.
del LCR y hay bandas oligoclonales de IgG. En el suero se han
hallado títulos de anticuerpos elevados. RESPUESTAS: 41: 1; 42: 4; 43: 5; 44: 3; 45: 4.

1653
Capítulo VII

NEOPLASIAS
DEL SISTEMA
NERVIOSO CENTRAL

Indice
Metástasis cerebrales Tumores de la médula espinal
Tumores primarios del SNC

Dr. JAVIER PARDO MORENO

METASTASIS CEREBRALES noma, los tumores del tubo digestivo y los cánceres pulmona-
res tienden a ser relativamente resistentes a las dosis de ra-
Los tumores más frecuentes del SNC son las metástasis. diación que tolera el SNC. El tratamiento de las metástasis
Aparecen aproximadamente en el 25% de los pacientes con leptomeníngeas requiere una combinación de radioterapia y
cáncer sistémico. quimioterapia intratecal.
El melanoma es el tumor con máxima probabilidad de dise-
minación al SNC. Los tumores de pulmón y mama tienen una TUMORES PRIMARIOS DEL SNC
prevalencia mayor y son responsables del mayor porcentaje de
metástasis en el SNC. Astrocitoma maligno
La extirpación quirúrgica de una metástasis cerebral única
El astrocitoma maligno o glioblastoma supone el 75% de los
puede mejorar la calidad de vida del paciente canceroso, de-
tumores gliales adultos. La biopsia con resección de las masas
pendiendo de la naturaleza y estadio de la enfermedad sisté-
accesibles, combinada con la radioterapia, mejora la supervi-
mica. La radioterapia es frecuentemente paliativa y puede de-
vencia, pero los logros son a corto plazo.
terminar mejorías neurológicas. Desafortunadamente el mela-
Meningioma
Los meningiomas son el 20% de los tumores encefálicos.
Nota: este tema pretende ser un complemento al tema de Son más frecuentes en las mujeres. Sus localizaciones más
Neurocirugía “Tumores del SNC” frecuentes son la hoz del cerebro, la convexidad cerebral late-

1654
NEUROLOGIA

ral, surco olfatorio, ala esfenoidal, tubérculo selar, agujero


25
magno y tentorio del cerebelo. Según sus características mi- 46
croscópicas se clasifican en: Los tumores más frecuentes del SNC son:

— Sincitiales. 1. Metástasis.
— Transicionales. 2. Astrocitomas.
3. Meningiomas.
— Fibroblásticos. 4. Neurinomas.
— Microquísticos. 5. Hemangioblastomas.
— Psamomatosos.
— Angioblásticos.
— Malignos.
47
La presentación clínica refleja la expansión lenta del tumor
¿Cuál es el tumor con mayor probabilidad de diseminación al SNC?:
con déficit neurológicos que evolucionan durante muchos años.
La TC permite observar calcio en el interior del tumor y la RM 1. Coriocarcinoma.
muestra una masa isodensa que capta gadolinio. El principal fac- 2. Epidermoide de pulmón.
tor determinante del tratamiento es la localización del tumor, ba- 3. Adenocarcinoma de mama.
rajándose entre las opciones la cirugía y la radioterapia. 4. Melanoma.
5. Carcinoma gástrico.
Schwannomas
Los schwannomas crecen habitualmente a partir de los ner-
vios craneales en las proximidades de sus agujeros de salida. 48
La resección puede ser curativa en estos tumores benignos, ¿Cuál es la localización más frecuente en todos los tipos de tumores me-
además de en los ependimomas, oligodendrogliomas, heman- dulares?:
gioblastomas y astrocitomas de bajo grado. La radioterapia
1. Dorsal.
puede reservarse para casos de deterioro.
2. Cervical.
3. Lumbar.
Linfomas primarios del SNC 4. Sacra.
El linfoma primario del SNC puede ser multifocal, simulando 5. Lumbosacra.
metástasis o placas de desmielinización. El linfoma primario es
más frecuente en pacientes con SIDA, déficit de IgM o IgA o
inmunodeprimidos por otras causas. Tras tratamientos cortos
con glucocorticoides puede producirse una notable reducción 49
de la masa del linfoma primario del SNC. El curso habitual es ¿Cuál es el método diagnóstico de elección para los tumores intramedu-
la recidiva y acaban por ser resistentes al tratamiento. lares?:
1. TC columna.
TUMORES DE LA MEDULA ESPINAL 2. RM medular.
3. Arteriografía.
Corresponden al 25% de los tumores intracraneales. Se cla- 4. Mielografía.
sifican como: 5. Rx columna.

— Intramedulares (en el interior del parénquima de la


médula espinal).
— Extramedulares (o intradurales). 50
— Extradurales. Con respecto a los meningiomas (señale la correcta):

La localización más frecuente en todos los tipos de tumores 1. Suponen el 80% de los tumores del SNC.
medulares es la médula dorsal. 2. Son más frecuentes en hombres.
3. Su localización más frecuente es la médula.
Los tumores extramedulares (intradurales y epidurales) cau-
4. La presentación clínica es abrupta.
san síntomas de compresión de la médula espinal o de las raí- 5. El tratamiento depende de la localización.
ces nerviosas. La sintomatología inicial suele consistir en dolor
local en la espalda y parestesias, lo que se sigue de un déficit
sensitivo por debajo del nivel de dolor, debilidad y disfunción RESPUESTAS: 46: 1; 47: 4; 48: 1; 49: 2; 50: 5.
de vejiga e intestino.

1655
NEOPLASIAS DEL SISTEMA NERVIOSO CENTRAL

Los tumores intramedulares se extienden a través de varios datos se alterarán si el tumor infiltra las meninges. Los tumores
segmentos de la médula espinal. Un patrón frecuente de afec- primarios deben ser tratados con técnicas de microneurocirugía, y
tación es un déficit sensitivo disociado, con alteración en la por lo general es posible la resección completa. Los tumores
sensibilidad térmica y dolorosa en los segmentos donde se ori- intramedulares más frecuentes, ependimomas y astrocitomas,
gina el tumor, y conservación de la función sensitiva de las co- sólo pueden ser extirpados parcialmente, por lo que es proba-
lumnas posteriores. ble su recidiva.
La RM ha sustituido a todas las demás modalidades en el estu- Cuando se sospeche tumor epidural en un paciente con cán-
dio de los pacientes con tumores epidurales o subdurales a nivel cer sistémico, la intervención terapéutica precoz puede evitar
del conducto raquídeo. La mielografía tras la inyección subarac- una paraparesia progresiva con disfunción vesical y aparición
noidea de contrastes no crónicos suele ser un complemento de la de un déficit sensitivo. Se deben administrar de forma inme-
RM. En el LCR suele existir hiperproteinorraquia, normocelulari- diata dosis elevadas de glucocorticoides y habitualmente se
dad y normoglucorraquia, teniendo en cuenta que los dos últimos recomienda también la radioterapia.

1656
Capítulo VIII

ENFERMEDAD
DE LA NEURONA
MOT ORA
Indice
Introducción Características clínicas
Etiología Exámenes complementarios
Epidemiología Tratamiento
Formas hereditarias de la enfermedad

Dr. JAVIER PARDO MORENO

INTRODUCCION ETIOLOGIA

La enfermedad de la neurona motora (ENM), también llama- Desconocida.


da a veces como esclerosis lateral amiotrófica, es un trastorno EPIDEMIOLOGIA
esporádico, en el que la primera y segunda motoneurona dege-
neran y desaparecen. La incidencia es similar en todas las partes del mundo
El daño puede afectar: (1.5/105/año), salvo en la isla de Guam y la península de Kii del
Japón, donde es mucho más alta.
— A las células del asta anterior de la médula espinal La prevalencia es de 6/105. Hombres/Mujeres (1.5:1).
(atrofia muscular espinal). Normalmente el establecimiento es a partir de los 50 años.
— A los núcleos motores de los pares craneales en el Es fatal aproximadamente a los 4 años desde el estableci-
tronco del encéfalo (parálisis bulbar). miento, aunque algunos pacientes sobreviven más tiempo.
— Al tracto corticoespinal (esclerosis lateral primaria).
FORMAS HEREDITARIAS DE LA ENFERMEDAD
— Al tracto corticobulbar (parálisis pseudobulbar).
ENM familiar
Lo más frecuente es que haya una mezcla de los cuatro Aproximadamente el 5% de las formas de adultos son auto-
componentes, si no desde el principio, sí en el curso de la en- sómicas dominantes. En los casos familiares no hay diferencia
fermedad. con respecto al sexo.

1657
ENFERMEDAD DE LA NEURONA MOTORA

Enfermedad de Werdnig-Hoffman Las funciones corticales y los sentidos especiales siempre


están indemnes. Las vías sensitivas suelen estar preservadas.
Es la atrofia muscular espinal infantil aguda que es autosómi-
ca recesiva con establecimiento intraútero o en neonatos. Hay
EXAMENES COMPLEMENTARIOS
hipotonía muscular generalizada, atrofia y debilidad que lleva a
la muerte en el primer año de vida. Se debe a una degeneración
Hay que solicitar hemograma, VSG, función hepática, protei-
de las células del asta anterior de la médula espinal.
nograma y electroforesis, serología luética, función tiroidea, Rx
tórax y hexosaminidasa en jóvenes pacientes.
Síndrome de Kugelberg-Welander
En el EMG veremos amplia denervación con normal o mode-
Es la atrofia muscular espinal crónica de la infancia que se rado enlentecimiento de la velocidad de conducción sensitiva y
presenta en niños con atrofia muscular lentamente progresiva, motora. Deben estar afectados dos o más miembros por el pro-
nomalmente simétrica y proximal, debida a pérdida de las cé- ceso.
lulas del asta anterior de la médula y del tronco cerebral. Lo En el LCR podemos encontrar hiperproteinorraquia (aprox.
más frecuente es la incapacidad progresiva y la muerte, pero a 100 mg./dl.), pero suele ser normal.
veces la esperanza de vida es normal. La RM se debe realizar cuando la sospecha de mielopatía
Suele ser autosómica recesiva, pero puede ser dominante. cervical o radiculopatía espondilótica lumbar existe.
La biopsia muscular es necesaria en pacientes que sólo tie-
CARACTERISTICAS CLINICAS nen debilidad en neuronas motoras inferiores, particularmente
si el curso es prolongado, para excluir polimiositis crónica, de-
El establecimiento es insidioso y progresivo y puede empe- ficiencia de maltasa ácida, etc. Tanto en la ENM como en las
zar en las piernas o en los brazos, suele ser asimétrico y dis- entidades anteriormente mencionadas debe haber característi-
tal y menos frecuentemente debuta con disartria o disfagia. cas de denervación en la biopsia muscular, pero en el EMG la
Las fasciculaciones con frecuencia son evidentes en los mús- velocidad de conducción motora está casi siempre disminuida
culos afectos desde estadios tempranos e incluso antes de en estos trastornos.
que aparezca debilidad o de que el paciente lo note.
Calambres, cansancio o debilidad son síntomas de presenta- TRATAMIENTO
ción comunes.
La insuficiencia respiratoria por debilidad de músculos res- No existe tratamiento definitivo y las medidas deben ir en-
piratorios es común en estadios tardíos, aunque también pue- caminadas al tratamiento sintomático. Ej.: quinina para los ca-
de ser la forma de debut. lambres; anticolinérgicos si existe dificultad para tragar la sali-
El dolor es relativamente frecuente debido a los calambres va; tratamiento farmacológico y físico de la espasticidad si és-
musculares. ta es dolorosa; apoyo terapéutico y ocupacional, etc.
La musculatura extraocular y los esfínteres (salvo estreñi- Actualmente en ensayo el Riluzole que podría aumentar
miento tardío) son normales durante toda la enfermedad. la supervivencia en 6 meses.

1658
Capítulo IX

ESCLEROSIS
MULTIPLE

Indice
Enfermedades desmielinizantes: esclerosis múltiple Esclerosis múltiple

Dra. MARIA ROSARIO ANTON

ENFERMEDADES DESMIELINIZANTES: Personas que emigran en la infancia de una zona de alto riesgo
ESCLEROSIS MULTIPLE a otra de bajo riesgo estarían parcialmente protegidas.

Las enfermedades desmielinizantes se caracterizan por la Factores genéticos


destrucción local o en placas de las vainas de mielina del SNC.
Los familiares en primer grado de pacientes tienen un riesgo
El prototipo, y la más frecuente, es la esclerosis múltiple, que
8 veces mayor. La concordancia entre gemelos monocigóticos
es una enfermedad inflamatoria desmielinizante del SNC que
se aproxima al 25%. No se ha encontrado un patrón de heren-
cursa habitualmente en brotes y cuya prevalencia en nuestro
cia predecible.
medio es de 50 casos/100.000 habitantes.
Se ha comunicado una mayor asociación del antígeno HLA
DR2 y DQW1 y la presencia de esclerosis múltiple.
ESCLEROSIS MULTIPLE
Factores autoinmunes
Etiología
Las lesiones en placas de la esclerosis múltiple son simila-
Desconocida.
res a las de la encefalomielitis alérgica experimental (enferme-
Se postula que podía estar relacionada con un proceso au-
dad autoinmune inducida en animales mediante inmunización
toinmune, mediado por células T y desencadenado por algún
con mielina), en la que la causa de la enfermedad es la sensi-
factor ambiental, que conllevaría la desmielinización en suje-
bilidad de los linfocitos T a un solo antígeno conocido como
tos genéticamente susceptibles.
proteína básica de la mielina (PBM); en la EM no se ha podido
demostrar que ésta sea la proteína diana, aunque se observa
Epidemiología
mayor frecuencia de células T reactivas frente a ella, aún ma-
Suele comenzar en la tercera-cuarta décadas; más frecuente yor en el LCR; también se ha visto aumento de células autorre-
en mujeres (60%) y en zonas templadas que en tropicales. activas frente a otros tipos de proteínas.

1659
ESCLEROSIS MULTIPLE

Clasificación por movimientos voluntarios y espontáneos, estímulos senso-


riales y dolorosos, cambios de temperatura, hiperventilación,
Remitente
ansiedad...
Remitente-progresiva Hay tendencia a presentar 2 o 3 formas asociadas, suele
responder a carbamazepina y menos a la fenitoína.
Progresiva primaria
En un 80% de los casos comienza por brotes y con los años — Neuralgia del trigémino. Más frecuente en adultos jó-
aumentan los cuadros de brotes-remisiones. Cuando debuta venes. Puede ser bilateral.
tras los 40 años suele adoptar la forma progresiva. — Crisis epilépticas.
La supervivencia media es mayor de 30 años tras el inicio de — Disartria paroxística.
la sintomatología. — Picazón o prurito.
— Pérdida transitoria de la visión o fenómeno de Uthoff.
Manifestaciones clínicas — Signo de Lhermitte.10-40%. Sensación de descarga
eléctrica de segundos de duración desde el cuello a
Cualquier episodio de alteración del SNC puede ser el sínto- región lumbar, aparece de forma espontánea o al fle-
ma de inicio, bien aislado (45%), o en combinación (55%), pero xionar el cuello.
uno de los más frecuentes es la neuritis óptica, junto a los de-
rivados de afectación del tronco cerebral. Los síntomas se van Diagnóstico
acumulando en la fase de estado, predominando los motores, Se basa en gran parte en la anamnesis y la exploración
sensitivos y troncoencefálicos, pero aumentando claramente neurológica, pero en la mayoría de los casos será necesario
los esfinterianos y mentales. La fatiga es un síntoma habitual. realizar exploraciones complementarias, ninguna de ellas es
La neuritis óptica (40%) se manifiesta como una pérdida de específica para el diagnóstico. Los criterios diagnósticos
agudeza visual unilateral (raramente bilateral) que se instaura aceptados son los de Poser; determinan un diagnóstico de
en horas o días, con dolor en globo ocular, el defecto campi- probabilidades basado en la multifocalidad y multitemporali-
métrico típico es un escotoma central, más evidente para los dad y en el que siempre hay que descartar otras patologías;
colores rojo y verde. La papila puede presentar con el tiempo se precisan 2 episodios clínicos, con localización en 2 nive-
atrofia, pero al inicio es normal o con palidez en el polo tempo- les distintos del SNC y separados en el tiempo. Se define el
ral. Ante una neuritis óptica el 40% desarrollarán EM, normal- inicio de la EM cuando el primer síntoma puede ser atribuido
mente en los dos años siguientes y sobre todo si es bilateral. claramente a la enfermedad. No hay ningun síntoma o signo
Las lesiones de médula espinal que interrumpen el tracto corti- patognomónico.
coespinal producen signos de disfunción de neurona motora supe- El estudio del LCR puede ser normal (66%) o mostrar pleoci-
rior, si se afectan las columnas posteriores se alterará la sensibili- tosis discreta (<10 cel./mm3). El dato más característico es el
dad vibratoria y posicional. La afectación de sensibilidad termoal- aumento de IgG (80%) por secreción intratecal, en contraste
gésica es menos frecuente. Son típicos los síntomas de disfunción con las concentraciones relativamente normales de albúmina y
vesical e intestinal. Los varones suelen referir impotencia (hasta proteínas totales. El índice de IgG es >0.7 en el 60% de las
un 44%). La mielitis transversa también puede producirse. EM. La IgG aumentada, en electroforesis se fracciona en ban-
Tanto en el caso de la neuritis óptica o de lesión medular, das oligoclonales hasta en un 90%.
1/3 curan completamente, 1/3 de forma parcial y el otro tercio Los potenciales evocados visuales, de tronco y somato-
no mejoran. sensoriales están afectados en un 80% de los casos (todos o
La neuromielitis óptica o enfermedad de Devic cursa con alguno de ellos). En un 30-40% hay alteración de los poten-
neuritis óptica aguda con frecuencia bilateral, seguida o prece- ciales sin evidencia clínica de lesión, por lo que pueden con-
dida en horas o semanas de mielitis transversa, puede existir firmar lesiones sospechosas en varias localizaciones del
pleocitosis polinuclear y proteinorraquia. SNC.
Otras manifestaciones las constituyen la diplopía, la paráli- Los potenciales visuales tienen valor pronóstico en cuanto a
sis del VI par, disartria y disfagia, afectación de pares protube- la recuperación de la función visual tras neuritis ópticas.
ranciales (V,VII,VIII) con acúfenos y/o sordera, vértigo, nistag- La RM craneal y medular es la prueba más sensible para de-
mus, parálisis facial periférica y neuralgia del V. La lesión de tectar lesiones sugerentes de EM. Son lesiones periventricula-
conexiones cerebelosas puede provocar ataxia, habla escándi- res, tambien pueden aparecer en cualquier punto de la sustan-
da, temblor de intención, etc. En las lesiones extensas puede cia blanca, realzan con contraste (gadolinio). Se han propuesto
existir alteración psíquica predominando la demencia de tipo tres criterios de mayor especificidad:
subcortical, sobre todo en lesiones del cuerpo calloso.
— Lesiones periventriculares.
— Una lesión infratentorial.
Alteraciones paroxísticas y transitorias
— Lesiones mayores de 5 mm.
Aparecen hasta en un 10%. Son bruscas, breves y estereoti- — También es muy específica la presencia de una lesión
padas. Pueden ser el síntoma de inicio. Se pueden presentar en cuerpo calloso.

1660
NEUROLOGIA

Diagnóstico diferencial
25
Alteraciones neurológicas multifocales que cursan 51
en brotes ¿Cuál de los siguientes síntomas o signos no es típico de la esclerosis
múltiple?:
— Inflamatorias: LES, Sjögren, Behçet, PAN, vasculitis
del SNC. 1. Neuritis óptica.
2. Neuralgia del trigémino.
— Infecciosas: Lyme, brucelosis, mycoplasma, VIH..
3. Afasia motora.
— Granulomatosis: sarcoidosis, Wegener... 4. Incontinencia de esfínter urinario.
— Alteraciones vasculares como la endocarditis bacte- 5. Diplopía.
riana aguda con embolias recurrentes.

Las formas progresivas


52
— Mielopatía espondilótica cervical.
— Degeneración combinada subaguda de la médula. Sobre el uso de corticoides en la esclerosis múltiple es cierto que:
— Paraparesia espástica tropical. 1. Modifican la historia natural de la enfermedad.
— Mielopatía asociada al HTLV-1. 2. Disminuyen el número de posibles brotes futuros.
— Enfermedad de neurona motora... 3. Reducen la intensidad y duración de los brotes.
4. Implican un mayor riesgo de recaída a largo plazo.
Tratamiento 5. Están contraindicados en las enfermedades desmielinizantes.

En el tratamiento sintomático de los brotes se han emplea-


do glucocorticoides, que únicamente disminuyen la gravedad
de los síntomas y aceleran la recuperación. Puede emplearse 53
ACTH o prednisona, pero lo más aceptado son los bolos de me- Ante un paciente con sospecha de enfermedad desmielinizante tipo es-
til-prednisolona en ciclos cortos de 3-7 días. clerosis mútiple no está indicado:
Para prevenir los brotes se ha empleado la azatioprina. 1. Estudio de L.C.R.
El interferon beta es el único fármaco capaz de modificar la 2. Potenciales evocados multimodales.
historia natural de la enfermedad en la forma recurrentes-re- 3. Resonancia Magnética craneal.
mitentes, disminuyendo en un tercio el número de brotes y dis- 4. Estudio inmunológico.
minuyendo el tamaño de las lesiones en la R. M. craneal. 5. Estudio de la velocidad de conducción nerviosa sensitiva y mo-
En las formas progresivas se usa la ciclofosfamida o el me- tora.
totrexate a dosis elevadas.
Tratamiento sintomático:
54
— En la vejiga neurógena espástica los anticolinérgicos.
— En la espasticidad el baclofeno o recientemente la ti- En el diagnóstico diferencial de la esclerosis múltiple no incluiría:
zanidina. 1. Enfermedad de Behçet.
— En las disestesias dolorosas, neuralgia del trigémino y 2. Lupus eritematoso sistémico.
espasmos tónicos la carbamazepina o la fenitoína. 3. Paraparesia espástica tropical.
4. Enfermedad de Lyme.
5. Enfermedad de Binswanger.

55
En el estudio con resonancia magnética en la esclerosis múltiple no es-
peraría encontrar:
1. Lesiones en la médula espinal.
2. Lesiones en cuerpo calloso.
3. Lesiones en sustancia gris.
4. Lesiones en troncoencéfalo.
5. Lesiones en cerebelo.

RESPUESTAS: 51: 3; 52: 3; 53: 5; 54: 5; 55: 3.

1661
Capítulo X

EPILEPSIA

Indice
Definición Diagnóstico diferencial
Clasificación de las crisis Tratamiento
Etiología Toxicidad de fármacos anticomiciales
Patogenia Nuevos anticomiciales
Cuadro clínico

DEFINICION — Crisis parciales simples.


— Crisis parciales complejas. Existe afectación del nivel
Trastorno caracterizado por una alteración crónica, recidi- de conciencia.
vante y paroxística de la función neurológica, secundaria a un — Crisis parciales secundariamente generalizadas.
trastorno en la actividad eléctrica del cerebro.
Cada episodio de alteración funcional cerebral se denomina Crisis generalizadas primarias
crisis. — Crisis tónico-clónicas.
Es importante en el concepto de epilepsia los términos reci- — Crisis tónicas.
divante y paroxística. Las crisis aisladas y no recidivantes pue- — Crisis de ausencia.
den producirse en personas sanas por diferentes motivos y no — Ausencias atípicas.
puede aplicarse el diagnóstico de epilepsia. — Crisis mioclónicas.
Todas las crisis no son de tipo convulsivo (manifestaciones — Crisis atónicas.
motoras), otras pueden manifestarse por alteraciones sensiti- — Espasmos infantiles.
vas, cognitivas o emocionales.
Status epiléptico
CLASIFICACION DE LAS CRISIS
— Estatus tónico-clónico.
La clasificación se basa en el modo de inicio de las crisis — Estatus de ausencia.
(focal o generalizado), en su posible posterior diseminación y — Epilepsia parcial continua.
en la afectación del nivel de conciencia.
ETIOLOGIA
Crisis focales o parciales
La clasificación etiológica más práctica es la basada en gru-
Se inician en una zona de la corteza cerebral. Según la afec- pos de edad que puede orientar posteriormente una actitud
tación del nivel de conciencia se dividen en: diagnóstica.

1662
NEUROLOGIA

Lactantes (0-2 años)


25
— Hipoxia e isquemia perinatal. 56
— Traumatismos craneales intraparto. El diagnóstico de crisis epilépticas se basa fundamentalmente en:
— Infección aguda. 1. La exploración neurológica.
— Trastornos metabólicos (hipoglucemia, hipocalcemia, 2. La historia clínica.
déficit de piridoxina..). 3. Las pruebas de neuroimagen.
— Malformaciones congénitas. 4. La respuesta al tratamiento.
5. El electroencefalograma.
Niños (2-12 años)
— Idiopáticas.
— Infecciones agudas. 57
— Traumatismos. La causa más frecuente de crisis comiciales en el adulto joven es:
— Convulsiones febriles. Niños con edades comprendi-
das entre 3 meses y 5 años pueden presentar en un 2- 1. Enfermedades cerebrovasculares.
2. Traumatismos.
5% de los casos crisis en los procesos febriles. La
3. Infecciones.
probabilidad de que estos niños presenten un cuadro 4. Alcoholismo.
epiléptico son mínimas cuando las crisis febriles du- 5. Enfermedades hereditarias.
ran menos de 5 minutos, son generalizadas y no se
asocian con alteraciones en el EEG interictal ni con fo-
calidad neurológica.
58
Adolescentes (12-18 años) En las convulsiones secundarias a traumatismos es cierto que:
— Idiopáticas. 1. A mayor duración de la pérdida de conocimiento más riesgo de
— Traumatismos. padecer una crisis.
— Abstinencia de alcohol y otras drogas. 2. La aparición de crisis en las primeras 24 horas es un signo de
— Malformaciones arterio-venosas. mal pronóstico.
3. La penetración de la duramadre se asocia a mayor incidencia
de crisis.
Adultos-jóvenes (18-35 años)
4. La aparición de una crisis tras un traumatismo craneal implica
— Traumatismos. el diagnóstico de epilepsia.
— Alcoholismo. 5. Las crisis post-traumáticas son más frecuentes en los pacientes
— Tumores cerebrales. Mayor incidencia en tumores de ancianos.
crecimiento lento.

Adultos (>35 años) 59


— Tumores. La diferencia entre crisis parciales simples y complejas se establece por:
— Enfermedades cerebro-vasculares. Generalmente se-
1. Alteración del nivel de conciencia.
cuelas tardías de embolias, hemorragias o trombosis,
2. Duración de las crisis.
ocasionalmente de forma aguda. 3. Aparición de sintomatología psíquica.
— Trastornos metabólicos. 4. Aparición de período de confusión post-crisis.
— Alcoholismo. 5. La edad del paciente.

• En los traumatismos,la penetración de la durama-


dre o la amnesia post-crítica> 24 horas se asocia 60
con mayor incidencia de crisis. La aparición de cri-
Sobre el status epiléptico es falso:
sis en las primeras 24 horas después del trauma-
tismo no constituye un pronóstico desfavorable. 1. Constituye una urgencia médica.
2. Puede deberse a un cambio en la medicación antiepiléptica.
PATOGENIA 3. Siempre se acompaña de pérdida de conciencia.
4. Puede producir deterioro orgánico irreversible.
5. En casos no controlables puede recurrirse al coma barbitúrico.
Las crisis pueden producirse por alteraciones metabólicas o
por lesiones anatómicas. La alteración primaria es una descar-
ga rítmica y repetitiva, hipersincrónica de muchas neuronas lo- RESPUESTAS: 56: 2; 57: 2; 58: 3; 59: 1; 60: 3.
calizadas en una zona del cerebro.

1663
EPILEPSIA

Se postulan al menos tres mecanismos por los que las crisis mente aparece una fase tónica con contracción muscular, pér-
pueden desarrollarse (por separado o en conjunción): dida de conciencia y del tono postural y en ocasiones un grito
(debido a la espiración forzada por la contracción de músculos
— Disminución de los mecanismos inhibitorios, sobre to- respiratorios).
do los producidos por el GABA. Posteriormente aparece una fase clónica caracterizada por
— Incremento de los mecanismos excitadores sinápticos. contracciones rítmicas musculares, finalmente aparece una fa-
— Incremento de las descargas neuronales endógenas. se de confusión postictal, con cefalea, amnesia del episodio,
somnolencia. Es típico de las crisis tónico-clónicas la mordedu-
CUADRO CLINICO ra de lengua y la relajación de esfínteres.
EEG: actividad rápida (10 Hz) de bajo voltaje en la fase tóni-
Crisis parciales o focales ca, posteriormente ondas más lentas y puntiagudas en ambos
Se inician con la activación de neuronas situadas en una zo- hemisferios. En la fase clónica brotes de ondas puntiagudas.
na de la corteza. El EEG interictal puede ser normal.
Se clasifican en simples cuando no se altera la conciencia ni Debe realizarse el diagnóstico diferencial con las crisis foca-
la percepción del ambiente y en complejas cuando sí existe di- les secundariamente generalizadas.
cha alteración.
Crisis tónicas
Crisis parciales simples Postura rígida de extremidades y/o tronco, generalmente
Crisis motoras con desviación de la cabeza y los ojos hacia un lado. Duración
menor que las tónico-clónicas.
Contracciones recurrentes de los músculos de una parte del
cuerpo sin pérdida de conciencia. En algunos casos las contrac- Crisis de ausencia (petit mal)
ciones van extendiéndose a lo largo de una extremidad siguiendo
la representación somatotópica cortical (evolución jacksoniana). Interrupción súbita de la actividad consciente sin actividad
muscular convulsiva ni pérdida del control postural. Duración
Crisis sensitivas de segundos o minutos. Pueden aparecer automatismos. No
existe período de confusión posterior.
Parestesias, sensaciones vertiginosas, alucinaciones auditi- EEG: descargas de espigas y ondas de 3 Hz (típico). El EEG
vas o visuales simples. interictal es normal.
El diagnóstico diferencial se realiza con crisis parciales com-
Crisis vegetativas plejas. En las crisis de ausencia no hay aura, la recuperación es
Crisis con sintomatología psíquica inmediata y el patrón del EEG es típico.
EEG: descargas regulares en espigas . Ausencias atípicas
Crisis parciales complejas Coexisten con otras formas de crisis generalizadas.
Se pierde el contacto con el ambiente. EEG: descargas de espigas y ondas de 2 a 4 Hz.
Se pueden iniciar con aura (corresponde a una crisis parcial Síndrome de Lennox-Gastaut. Asociación de crisis: atónicas,
simple): fenómeno de dejà vu, tensión emocional, alucinaciones... mioclónicas y ausencias atípicas. Se asocia con retraso inte-
Durante la crisis puede interrumpirse la actividad que se lectual.
realiza en el momento, o pueden aparecer automatismos. EEG: punta-onda lenta y ritmos reclutantes en el sueño.
EEG: espigas uni-bilaterales o descargas lentas en región
temporal o témporo-frontal, tanto en período de crisis como in- Crisis mioclónicas
terictal. Frecuentemente se detecta actividad en hipocampo y Contracciones musculares súbitas, breves, aisladas o repeti-
amígdala. tivas, que pueden afectar a parte o a todo el cuerpo, condicio-
nando entonces una caída al suelo. No existe pérdida de con-
Crisis parciales secundariamente generalizadas ciencia.
La presencia de un aura, signo de focalidad o signo defici- EEG: ondas y poliespigas u ondas lentas y puntiagudas.
tario focal postictal nos indicará un inicio focal. Las crisis mioclónicas pueden deberse a diversas causas:

Crisis generalizadas primarias — Idiopáticas.


— Secundarias: uremia, insuficiencia hepática, Creutz-
Crisis tónico-clónicas (grand mal)
feldt-Jackob, enfermedad de Lafora, etc.
Se suelen preceder de una fase prodrómica consistente en — Epilepsia mioclónica juvenil. Adolescentes, alteración
sensaciones premonitorias de que algo va a suceder, posterior- en el cromosoma 6. Convulsiones mioclónicas al des-

1664
NEUROLOGIA

pertar que se transforman en crisis tónico-clónicas ge-


25
neralizadas. 61
En el diagnóstico diferencial de las crisis comiciales no incluiría:
Crisis atónicas
1. Cuadros histéricos.
Pérdidas breves de conciencia y del tono postural que no se 2. Cuadros de migraña.
asocian con contracciones musculares tónicas. Son típicas de 3. Cuadros sincopales.
la infancia. 4. Accidente isquémico transitorio.
EEG: poliespigas y ondas lentas. 5. Brote de esclerosis múltiple.
Se realizará el diagnóstico diferencial fundamentalmente con
el síncope, la cataplejía y el accidente isquémico transitorio.

Espasmos infantiles o en flexión (Sdr. de West) 62


Generalmente en lactantes menores de 12 meses. No emplearía en el tratamiento del status:
Contracciones breves y sincrónicas de músculos del cuello, 1. Difenilhidantoína en perfusión continua i.v.
tronco y extremidades superiores (en flexión). Suele coexistir 2. Barbitúricos.
un trastorno neurológico: encefalopatía anóxica, esclerosis tu- 3. Medidas de soporte ventilatorio.
berosa, etc. 4. Carbamacepina a altas dosis.
Se asocia retraso mental en un 90% de los casos. 5. Diazepam en bolo i.v.
El tratamiento se basa en ACTH o ácido valproico.

Status epiléptico
Crisis muy prolongadas o que se repiten sin período de recu- 63
peración entre las mismas. El diagnóstico diferencial fundamental de las crisis de ausencia es con:
El status tónico-clónico es una urgencia médica, ya que pue-
1. Cuadro sincopal.
de producir la muerte o un deterioro orgánico irreversible.
2. Crisis parciales secundariamente generalizadas.
Puede desencadenarse por un cambio o supresión brusca de la 3. Crisis parciales complejas.
medicación, cese brusco en la ingesta de alcohol, traumatis- 4. Crisis parciales simples.
mos, fiebre.. 5. Crisis atónicas.
En el status de ausencia no hay pérdida de conocimiento ni
trastornos convulsivos.
La epilepsia parcial continua es frecuentemente con mani-
festaciones motoras, pero tambien puede ser sensitiva o visce- 64
ral. Pueden aparecer automatismos muy complejos.
La aparición de hiperplasia gingival e hirsutismo son efectos secundarios
DIAGNOSTICO DIFERENCIAL típicos del uso de:
1. Acido valproico.
Accidente isquémico transitorio 2. Carbamacepina.
3. Lamotrigina.
Déficit neurológico generalmente menor de 1 hora y en todo 4. Felbamato.
caso menor de 24 horas. Es más prolongado que una crisis, ra- 5. Difenilhidantoína.
ra vez convulsivo, no existe pérdida de conciencia. El problema
aparece en mayores de 60 años, donde la etiología vascular de
la epilepsia es mayor del 40%.

Jaqueca 65
La migraña acompañada o migraña con aura puede confun- En el diagnóstico diferencial de las crisis es cierto que:
dirse con crisis parciales simples, especialmente aquellas con 1. En los pacientes migrañosos el EEG es normal.
sintomatología somatosensorial, visual, ilusorias o alucinato- 2. Las migrañas con aura simulan crisis parciales complejas.
rias. Además en un 6% de las migrañas aparecen alteraciones 3. La prolactina sérica es normal después de una crisis.
del EEG. 4. Las crisis atónicas deben diferenciarse de la cataplejía.
5. En las crisis histéricas existe pérdida de conciencia.
Histeria
No suele ser nocturna. Presencia de testigos. Relación RESPUESTAS: 61: 5; 62: 4; 63: 3; 64: 5; 65: 4.
con conflictos personales. Crisis muy aparatosas. No pérdida

1665
EPILEPSIA

de conciencia. Pueden responder a la sugestión. No confu- — Iniciar perfusión continua de fenitoína 18 mg./kg.
sión post-crisis. Indiferencia anta la enfermedad. La prolacti- — En casos no controlables recurrir a la anestesia con
na después de las crisis es normal (en las crisis reales au- barbitúricos o anestésicos inhalados.
menta)
TOXICIDAD DE FARMACOS ANTICOMICIALES
TRATAMIENTO
— Difenilhidantoína: signos cerebelosos, hiperplasia gin-
Crisis generalizadas tónico-clónicas gival, hirsutismo, exantema cutáneo.
Difenilhidantoína, carbamazepina, fenobarbital. ácido val- — Carbamazepina: ataxia, mareos, mielosupresión, he-
proico. patotoxicidad.
— Fenobarbital: exantema, disminución de la libido, ma-
Crisis parciales reos, confusión.
— Acido valproico: ataxia, temblor, hepatotoxicidad,
Difenilhidantoína o carbamazepina. mielosupresión.
Ausencias simples NUEVOS ANTICOMICIALES
Acido valproico o etosuximida.
— Gabapentina. Indicado en crisis parciales y secunda-
Ausencias atípicas riamente generalizadas.
— Lamotrigina. Indicado en crisis parciales, secundaria-
Acido valproico o clonazepam. mente generalizadas, mioclonías y ausencias.
— Felbamato. Crisis atóricas, parciales, secundariamente
Status epiléptico generalizadas, Sdr. de Lennox-Gastarut.
El status es una emergencia médica. — Vigabatrina. Crisis parciales, Sdr. de West.

— Protección de vía respiratoria y lengua. Características comunes: se les puede monitorizar niveles
— Glucosa al 50% bolo de 50 ml. sanguíneos, se usan cuando falle la mono o politera-
— Infusión con salino y vitamina B1. pia con los clásicos. Pocas interacciones farmacológi-
— Diazepam i.v. 2 mg/min hasta máximo de 20 mg. cas.

1666
Capítulo XI

CEFALEAS

Indice
Definición Cefalea de tipo tensional o de estrés
Anatomía Clasificación y diagnóstico diferencial según curso
Migraña temporal
Cefalea histamínica «en racimos» o de Horton

Dra. MARIA ROSARIO ANTON

DEFINICION res u otros procesos expansivos, presión intracraneal elevada,


espasmo muscular y causas psíquicas.
Se refiere a toda molestia o dolor localizado en la cabeza
(bóveda craneal). MIGRAÑA
Ante un paciente con cefalea lo más importante es realizar
una buena historia clínica: carácter de la cefalea (punzante, Cefalea episódica familiar (>60%), más frecuente en muje-
opresivo...), intensidad, localización, modo de comienzo, fre- res (3:1), de inicio en infancia o adolescencia. Afecta al 20-
cuencia, duración, momento del día, síntomas acompañantes, 30% de la población.
factores precipitantes y aquellos que la exacerban o alivian. Asintomático entre crisis, éstas pueden iniciarse por un pró-
dromo y precederse por un aura; el aura puede ocurrir sola, pe-
ANATOMIA ro en general se sigue de cefalea. La cefalea es hemicraneal,
dura 4-24 horas, continua o pulsátil. Son típicos la fono y foto-
Las estructuras craneales sensibles al dolor son la piel, el fobia, náuseas, vómitos, hipersensibilidad al movimiento y al
tejido celular subcutáneo, los músculos, arterias, periostio, Valsalva.
mucosas, senos venosos, parte de la dura de la base craneal y Como factores desencadenantes: chocolate, alcohol, luz bri-
las arterias meníngeas. llante, leche, café, té, glutamato monosódico (comida china), hi-
El dolor de cabeza está representado por la inervación sen- poglucemia, ejercicio excesivo, tensión, anticonceptivos orales.
sitiva de los pares craneales V, IX y X y los nervios cervicales En ocasiones precede o acompaña a la menstruación y suele
C1, C2 y C3. mejorar con la menopausia. Empeora en el primer y mejora en
Los factores que causan cefalea son: la inflamación de las el segundo y tercer trimestre de gestación.
estructuras con receptores del dolor, dolor referido, irritación Según la presencia de aura se clasifica en:
meníngea, tracción o dilatación de los vasos sanguíneos, pre-
sión o distorsión de estructuras sensitivas causado por tumo- — Migraña sin aura o común.

1667
CEFALEAS

— Migraña con aura o clásica. — En migraña severa refractaria se puede usar dexame-
tasona y después ciclos cortos de prednisona.
Existen diferentes tipos de aura: — Sumatriptán, es un agonista de receptores de seroto-
nina 1D, disponible en forma oral y subcutánea, se ha
— Alteración visual homónima. mostrado más efectivo que los tratamientos antes co-
— Parestesia y/o acorchamiento unilateral. mentados pero con mayor tasa de recurrencia.
— Debilidad unilateral.
— Afasia o dificultad para el habla. CEFALEA HISTAMINICA,”EN RACIMOS”,
O DE HORTON (Cluster headache)
La más frecuente es el aura visual: visión en empalizada, fo-
topsias, etc. No familiar, más frecuente en varones (4:1), dolor orbitario
Pueden existir síntomas neurológicos que no se acompañan constante, unilateral, intenso, con congestión nasal y lagrimeo,
o se siguen de cefalea: equivalentes migrañosos (en niños los suele aparecer 2-3 horas después de dormirse (frecuente en
vómitos, vértigo paroxístico... Dolor torácico, acúfenos, altera- fase REM), dura 30-90 minutos, ocurre varias veces al día du-
ciones visuales..). rante semanas o meses y luego desaparece, puede reaparecer
La migraña basilar (Bickerstaff) presenta alteración visual y en época de estrés o desencadenarse por alcohol, nitratos o
parestesias bilaterales, confusión, estupor, agresividad, diplo- alimentos con tiramina. No se precede de aura, pródromos, ni
pía y disartria, se presenta de forma parcial en el 30% de ni- hay síntomas gastrointestinales.
ños con migraña. El dolor cede con sumatriptán y esteroides, y se puede em-
La migraña oftalmopléjica requiere diagnóstico diferencial plear verapamil como agente profiláctico.
con aneurisma carotídeo (arteriografia).
CEFALEA DE TIPO TENSIONAL O DE ESTRES
Tratamiento
Es la más frecuente en todas las edades, sobre todo en la
Tratamiento profiláctico
edad media.
El más importante. Cefalea bilateral, profunda, sorda, ”en banda”, difusa o de
localización occipital o bifrontal, leve-moderada. Empeora a lo
— Evitar factores desencadenantes o agravantes. largo del día, puede asociar anorexia, oto-fonofobia y contrac-
tura de músculos pericraneales. No se agrava con el ejercicio.
Si hay más de 2-3 crisis al mes, se inicia tratamiento farma- Los analgésicos son poco eficaces, y los pacientes pueden
cológico: caer en la automedicación en exceso. Es el unico tipo de cefa-
lea “absolutamente” continua. Puede ser episódica (<15 días
— Betabloqueantes (propranolol), más indicados en pa- al mes) o crónica (más de 15 días al mes).
cientes con estrés, angor o hipertensión. Disminuyen Muchos pacientes sufren exacerbaciones agudas que cum-
la frecuencia e intensidad. plen criterios de migraña: cefalea mixta vásculo-tensional.
— Antagonistas del calcio. Reducen sólo la frecuencia.
Utiles en hipertensos, aura prolongada y migraña ba- CLASIFICACION Y DIAGNOSTICO
silar. DIFERENCIAL SEGUN CURSO TEMPORAL
— Antagonistas serotoninérgicos. En la migraña con aura.
— Antidepresivos tricíclicos son efectivos en pacientes Cefalea simple aguda
con cefalea tensional.
— Hemorragia subaracnoidea. Cefalea relacionada con
el ejercicio puede ser el signo de ruptura de un aneu-
Tratamiento sintomático
risma.
— Analgésico + cafeína o codeína o barbitúrico. Poten- Suele ser una cefalea sobreaguda, severa, con rigidez
cial adicción y propensión a producir cefalea. de nuca, náuseas, vómitos, signos de focalidad neuro-
— Narcóticos (codeína, meperidina, dextropropoxifeno...) lógica y a veces confusión y coma. Ante la sospecha
en cefaleas refractarias, gestación. realizar TAC craneal y si no hay signos de hemorragia
— Asociar al analgésico ansiolíticos, antieméticos o neu- punción lumbar.
rolépticos. — Meningitis. Cefalea severa con fiebre y signos menín-
— AINEs (el más usado el naproxen) no originan depen- geos.
dencia. — Sinusitis purulenta aguda. Fiebre y dolor localizado. El
— La ergotamina y dihidroergotamina son efectivas, pero diagnóstico es difícil en la sinusitis esfenoidal.
tienen mala absorción oral, originan cefalea de rebote — Neuritis óptica. Asocia dolor orbitario y disminución
y dependencia. de agudeza visual.

1668
NEUROLOGIA

— Otros procesos: glaucoma agudo, en hipertensión se-


25
vera, disección espontánea de carótida interna (se 66
sospecha ante un síndrome de Horner doloroso), cefa- La cefalea más frecuente en todas las edades es:
lea post punción lumbar que aparece a las 24-48 ho-
ras, dura 4-7 días, frontal, postural, cede al tumbarse 1. Migraña con aura.
y predomina en mujeres. Se relaciona con el grosor de 2. Migraña sin aura.
3. Cefalea tensional.
la aguja empleada.
4. Cefalea en racimos.
5. Cefalea por abuso de ergotamínicos.
Cefalea recurrente aguda
— Migrañas o cefaleas tensionales.
— Hidrocefalia intermitente.
— Feocromocitoma. Crisis hipertensivas que asocian ce- 67
falea pulsátil, intensa y de corta duración. Se denomina migraña de Bickerstaff a:
— Síndrome carcinoide.
— Neuralgia del trigémino. Paroxismos de dolor breve, 1. Migraña basilar.
como una descarga eléctrica, con zonas “gatillo” de- 2. Migraña con aura.
3. Migraña oftalmopléjica.
sencadenantes. Responde a carbamazepina.
4. Cefalea histamínica.
— Cefalea en racimos. 5. Migraña sin aura.
— Hipertensión intracraneal idiopática.Puede presentar-
se con/sin papiledema, afecta a mujeres jóvenes.

Cefalea subaguda
68
Se presenta en días o semanas, puede ser el comienzo de La cefalea en racimos (señale la falsa):
una cefalea tensional crónica pero generalmente representa
una alteración neurológica primaria: tumores intracraneales, 1. Es más frecuente en varones.
hematomas subdurales, abscesos cerebrales, trombosis de se- 2. El dolor cede con sumatriptán y esteroides.
3. Puede desencadenarse con el alcohol.
nos venosos, arteritis de células gigantes en el anciano.
4. Suele aparecer 2-3 horas después de dormirse.
La arteritis de células gigantes es más frecuente entre 60- 5. Es siempre bilateral.
80 años, asocia cefalea con síntomas constitucionales, dismi-
nución de agudeza visual y más raramente oftalmoplejía o ic-
tus.
Investigar sobre datos de polimialgia reumática, existe hi-
persensibilidad en región temporal, ausencia de pulso, claudi- 69
cación mandibular. VSG aumentada. En la evolución hay cefa- Entre las siguientes entidades , ¿cuál se asocia a polimialgia reumática?:
lea en el 60-100%, sigue el trayecto de la arteria temporal, 1. Neuralgia del trigémino.
pulsátil, opresivo, aparece en horas o días. Responde a este- 2. Feocromocitoma.
roides, deben ser administrados lo más precozmente posible. 3. Hidrocefalia intermitente.
4. Arteritis de células gigantes.
Cefalea crónica no progresiva 5. Hipertensión intracraneal idiopática.
Puede asociarse a cefalea tensional, sobreabuso de analgé-
sicos, tumores, alteraciones psiquiátricas, etc.
La ingesta diaria de antipiréticos, analgésicos o productos
con ergotamínicos sedantes o hipnóticos puede provocar una 70
cefalea crónica diaria. Desaparece tras un mes de suspender ¿Cuál de estas cefaleas se presenta habitualmente de manera sobreagu-
el fármaco. Se puede emplear profilaxis con betabloqueantes da?:
y/o antidepresivos tricíclicos.
1. Hemorragia subaracnoidea.
2. Cefalea tensional.
3. Cefalea por abuso de analgésicos.
4. Arteritis de células gigantes.
5. Cefalea secundaria a absceso cerebral.

RESPUESTAS: 66: 3; 67: 1; 68: 5; 69: 4; 70: 1.

1669
Capítulo XII

PAT OLOGIA MEDULAR

Indice
Anatomía Formas clínicas
Semiología medular

ANATOMIA Las raíces lumbares, sacras y la coxígea atraviesan mucho


espacio subaracnoideo, constituyendo la cola de caballo.
La médula espinal se extiende desde el límite superior del
atlas hasta el borde inferior de L1. Superiormente se continúa Aferencias
con el bulbo e inferiormente con el filum terminale, que llega
Sensibilidad protopática
hasta el coxis. La longitud de la médula es de 45 cm. y la co-
lumna vertebral mide 70 cm. Dolor y temperatura: Fibras mielínicas y amielínicas finas
que hacen sinapsis en la 2.a neurona situada en el asta dorsal,
Localización en superficie las fibras se decusan en la comisura blanca anterior y se diri-
gen al fascículo espinotalámico lateral.
— Las apófisis espinosas cervicales marcan el corres-
pondiente segmento medular.
Tacto: Las fibras siguen el mismo trayecto que las anterio-
— Las de la región torácica alta marcan dos niveles infe- res pero se localizan en el fascículo espinotalámico anterior.
riores, así la apófisis espinosa de T4 se palpa en un
Sensibilidad epicrítica
nivel correspondiente a T6 medular.
(discriminativa, peso, posicional, etc.)
— Las de región torácica baja marcan tres niveles infe-
riores. Fibras mielínicas gruesas, ingresan en el asta dorsal medu-
lar y se dirigen al cordón posterior, por donde ascienden hasta
La médula es una estructura anatómicamente continua de la los núcleos grácil y cuneiforme (2.a neurona) del bulbo, donde
que emergen 31 pares de nervios. De cada segmento medular después de hacer sinapsis se decusan.
emergen dos raíces anteriores y dos raíces posteriores, cada
raíz anterior y posterior se unen formando los nervios radicula- Eferencias
res: 8 cervicales, 12 torácicos, 5 lumbares, 5 sacros y uno coxí-
Haz corticoespinal
geo.
La primera raíz cervical sale por encima de C1, la 8.a raíz por Desde las áreas corticales 4 y 6 (también las áreas 1, 2 y 3)
debajo de C7, a partir de ahí cada raíz sale por debajo de la parten fibras nerviosas motoras que atraviesan la corona ra-
vértebra homónima. diata, la cápsula interna (brazo posterior) y la parte ventral del

1670
NEUROLOGIA

bulbo y la protuberancia, en el bulbo forman las pirámides y en


25
la parte inferior del mismo se decusan (el 90% de las fibras) 71
para constituir el haz corticoespinal lateral. El otro 10% des- Es incorrecto respecto a la siringomielia:
ciende formando el haz corticoespinal directo o anterior, que
se decusa en la comisura blanca anterior (sus fibras se dirigen 1. Se asocia a tumores medulares.
a grupos musculares de región cervical). 2. Se puede acompañar de parálisis de cuerdas vocales e hipotro-
fia lingual.
3. Se asocia a malformaciones cráneo-cervicales.
Vascularización medular 4. Se caracteriza por pérdida de sensibilidad dolorosa y posicional
Tres ejes arteriales: con preservación de sensibilidad táctil y algésica.
5. Una intervención quirúrgica descompresiva puede mejorar el
cuadro.
— Arteria espinal anterior. Irriga los dos tercios anterio-
res de la médula espinal.
— Dos arterias espinales posteriores.
72
Existen zonas más vulnerables a la isquemia, en concreto la En las tabes dorsal es típico:
más vulnerable es la región tóraco-lumbar irrigada por la arte-
1. Es una manifestación precoz de la sífilis.
ria de Adamkiewick. 2. Se asocia con la pupila tónica de Adie.
3. El tratamiento es quirúrgico.
SEMIOLOGIA MEDULAR 4. Existe pérdida de sensibilidad posicional y vibratoria.
5. Existe una marcha pareto-espástica.
La patología medular se distingue por la existencia de dife-
rentes niveles que nos ayudan a localizar la lesión.
73
Niveles sensitivos
Respecto a la degeneración combinada subaguda medular no es cierto
— T5-mamas. que:
— T10-ombligo. 1. Existe un patrón de anestesia “suspendido”.
2. Existe un déficit de vitamina B12.
Niveles musculares 3. Presencia de paraparesia espástica.
4. Presencia de ataxia progresiva.
— Niveles superiores a C3-C4 producen parálisis diafrag- 5. Se alteran fundamentalmente las columnas posterolaterales.
mática.
— La inervación de EESS en niveles superiores a T1.
— La inervación de EEII en niveles superiores a L1.
74
Niveles reflejos En un síndrome de hemisección medular no esperaría encontrar:
— Reflejos superficiales. 1. Hipoestesia termoalgésica contralateral debajo de la lesión.
2. Hemiparesia espástica contralateral por debajo de la lesión.
• Cutáneo-abdominales superiores: T6-T9. 3. Disminución de sensibilidad propioceptiva ipsilateral por deba-
• Cutáneo-abdominales medios: T9-T11. jo de la lesión.
• Cutáneo-abdominales inferiores: T11-L1. 4. Alteraciones de segunda motoneurona al nivel de la lesión.
• Cremastérico: L1-L2. 5. Presencia de signo de Babinski en miembro inferior ipsilateral a
la lesión.
— Reflejos miotáticos.
• Biccipital: C5
• Triccipital: C7 75
• Estilorradial: C6 En la esclerosis lateral amiotrófica la lesión se sitúa en:
• Rotuliano: L4
1. Cordones posteriores.
• Aquíleo: S1 2. Raíces anteriores y ganglios raquídeos.
3. Asta anterior medular y vía piramidal.
FORMAS CLINICAS 4. Astas intermedio laterales y raíces anteriores.
5. Cordones posterolaterales.
Sección completa medular
Se produce alteración de todas las funciones (motoras y RESPUESTAS: 71: 4; 72: 4; 73: 1; 74: 2; 75: 3.
sensitivas) por debajo del nivel lesional.

1671
PATOLOGIA MEDULAR

Etiología La siringomielia puede ser idiopática o secundaria a tumo-


res, traumatismos, compresión extrínseca, aracnoiditis...
— Traumatismos.
La forma idiopática se asocia a otras malformaciones:
— Tumores (linfoma, metástasis).
Arnold-Chiari, mielomeningocele, platibasia, Dandy-Walker.
— Esclerosis múltiple.
— Alteraciones vasculares.
Alteraciones de columnas posterolaterales
Clínicamente Etiología
— Abolición de todas las modalidades sensitivas por de- — Degeneración subaguda combinada de la médula por
bajo de la lesión. déficit de vitamina B12.
— Tetraplejía o paraplejía. Inicialmente parálisis flácci- — Mielopatía vacuolar asociada al VIH.
da, arrefléxica por shock espinal, posteriormente hi- — Compresión extrínseca.
pertónica e hiperrefléxica.
— A nivel lesional, signos de segunda motoneurona. En la degeneración subaguda combinada de la médula por
— Alteraciones de la sudoración, alteraciones tróficas, al- déficit de vitamina B12 se produce una paraparesia espástica y
teraciones del control de esfínteres con incontinencia. atáxica progresiva con neuropatía y parestesias en manos y
pies.
Hemisección medular (síndrome Brown-Sequard)
Alteración de columnas posteriores
Etiología
Etiología
Típico de las lesiones extramedulares.
Tabes.
— Disminución de la sensibilidad termo-algésica contra- En la tabes dorsal el síntoma más frecuente es un dolor lan-
lateral por debajo de la lesión. cinante fugaz y repetitivo. Existe ataxia de extremidades infe-
— Disminución de la sensibilidad propioceptiva ipsilate- riores por pérdida de sensibilidad posicional. Los signos más
ral por debajo de la lesión. importantes son: pérdida de sensibilidad posicional y vibratoria
— Debilidad espástica ipsilateral por debajo de la lesión. en EEII, pérdida de reflejos en extremidades inferiores, signo
— Alteraciones de segunda motoneurona al nivel de la de Romberg, pupilas de Argyll-Robertson.
lesión.
Alteración de asta anterior
Alteraciones centrales
Etiología: Atrofias musculares espinales.
Etiología
Alteración de asta anterior y tracto piramidal
— Siringomielia.
— Siringobulbia. Etiología
— Tumores intra-medulares.
Esclerosis lateral amiotrófica.
— Se produce una disociación sensitiva: hemianestesia y
analgesia en un nivel suspendido bilateral y preserva- Síndrome de la arteria espinal anterior
ción del resto de sensibilidades. En la siringomielia es
típica la afectación a nivel medular cervical con anes- Afectación de cordón anterior + asta anterior + base de as-
tesia suspendida en ambas extremidades superiores. tas posteriores + área periependimaria.
— Cuando la lesión progresa puede aparecer afectación
de las células del asta anterior (paresia, fasciculacio-
nes), síndrome de Horner ipsilateral (C8-T1) y signos NOTA: La patología medular traumática, tumoral y compre-
piramidales. siva se expone en la sección de neurocirugía.
Señale cuál de las siguientes opciones no es característica de las reac-
ciones adversas a medicamentos tipo A:

1. Previsibles en función de las acciones del fármaco.


2. Debidas a un efecto exagerado.
3. Intensidad en relación directa con la dosis administrada.
4. Tratamiento con medias sintomáticas.

1672
Capítulo XIII

NEUROPATIAS
PERIFERICAS
Indice
Polineuropatia Mononeuropatía múltple
Mononeuropatía

Dra. MARIA CASTRO DEL RIO

POLINEUROPATIA — Fasciculaciones y calambres.


— Disminución o pérdida de sensibilidad distal, así como
Concepto parestesias, disestesia, hiperestesia, hiperpatía y ata-
Incluye todos los trastornos de los nervios motores, sensiti- xia sensitiva.
vos y autonómicos, excluyendo aquellos en los que se afecta — Alteraciones tróficas y deformidades.
un único nervio o más, aisladamente (Mononeuropatías). — Trastornos autonómicos: incluyen hipotensión postu-
ral o impotencia, anhidrosis, arritmias cardíacas...
Etiología
Exámenes complementarios
Se han descrito causas hereditarias, metabólicas, infeccio-
sas, colagenosis, deficiencias vitamínicas, toxinas (productos — Hemograma y bioquímica dirigida a descartar altera-
tóxicos y fármacos), enfermedades malignas... ciones tóxicas, metabólicas y nutricionales.
— Electrodiagnóstico: E.M.G y estudios de conducción
Fisiopatología nerviosa: Ante un proceso polineuropático generaliza-
do, es posible reconocer si se trata de un trastorno
Existen tres mecanismos de enfermedad del nervio periférico: axonal o desmielinizante: en los procesos desmielini-
zantes se enlentece pronto la velocidad de conducción
— Degeneración Waleriana. nerviosa; mientras, en los procesos axonales, la con-
— Degeneración axónica. ducción nerviosa es normal hasta una fase tardía.
— Degeneración segmentaria. — Biopsia de nervio: se emplea en caso de sospecha de
mononeuropatía múltiple; en la evaluación de polineu-
Clínica
ropatías desmielinizantes inflamatorias crónicas (se
— Debilidad muscular de predominio distal. suele biopsiar el N. sural) y en el diagnóstico de tras-
— Hiporreflexia o arreflexia. tornos pediátricos genéticamente determinados.

1673
NEUROPATIAS PERIFERICAS

Clasificación. Ver tabla I testinal leve de una a tres semanas antes del comienzo de la
clínica. Otros hechos descritos como antecedentes son las in-
Síndrome de Guillain-Barré
tervenciones quirúrgicas y enfermedades linfomatosas (sobre
Es una polirradiculoneuropatía aguda y simétrica, caracteri- todo la Enf. Hodgkin). La mayoría de datos sugieren que se tra-
zada por una debilidad predominantemente motora y de pro- ta de una reacción inmunológica mediada por células y dirigida
gresión ascendente. contra los nervios periféricos.

Epidemiología Clínica

La distribución geográfica es universal, afectando a todas Se caracteriza por debilidad que evoluciona simétricamente,
las edades y a los dos sexos. Su incidencia mundial es de 1- alcanzando su máximo antes de dos semanas (90%). Comienza
1.5/100.000/año. afectando musculatura proximal y distal de las extremidades;
habitualmente se afectan primero las extremidades inferiores,
después progresa a los músculos intercostales de tronco, ex-
Etiología
tremidades superiores y, finalmente, pares craneales (diplejía
Es desconocida; pero se ha relacionado con los siguientes facial en la mitad de casos).
hechos: un 60-70 % han sufrido una infección respiratoria o in- Otros datos son la hipotonía y arreflexia.

TABLA I
Clasificación de polineuropatías

AXONAL DESMIELINIZANTE

AGUDA Porfiria S. Guillain-Barré


Taxol (antineopl.) Carcinoma
Linfoma
Difteria
Sida (Primoinfecc.)

CRONICA D.M. , Uremia Polineuropatía desmielinizante


Déficit B12,EPOC Crónica.
Cirrosis biliar 1.a Diabetes mellitus
Amiloidosis 1.a Hepatopatía crónica
Ss. Malabsorción Hipotiroidismo
Carcinoma, Linfoma Plasmocitoma solitario
Policitemia vera o MM tipo osteoesclerótico
Mieloma M. Lítico Macroglobulinemia
Gammapatía Amiodarona
monoclonal benigna
tipo IgA e IgG
Crioglobulinemia
N. hereditarias
Fármacos: N. hereditarias:
— Vincristina E. Charcot-Marie-
— INZ -DPH Tooth
Alcohol E. Déjerine-Sotas
Organofosfatos E. Refsum

1674
NEUROLOGIA

Puede existir afectación de la sensibilidad (sobre todo la


25
profunda), pero la enfermedad es predominantemente motora. 76
También se producen alteraciones autonómicas (trastornos ¿Qué es incorrecto sobre el síndrome de Guillain-Barré?:
del ritmo cardíaco).
1. Es una polirradiculoneuritis aguda.
2. La disociación albúmino-citológica no ocurre en el 100% de los
Variantes casos.
3. El pronóstico empeora en la forma axonal del síndrome.
— Afectación de músculos craneales y de brazos antes 4. Los pares craneales no se afectan en esta enfermedad.
que los músculos de las piernas. 5. Una de las causas más frecuentes de muerte es la embolia de
— Síndrome de Miller-Fisher: Oftalmoplejía, ataxia y pulmón.
arreflexia.

Exámenes complementarios 77
¿Cuál de los siguientes criterios es imprescindible para el diagnóstico del
El LCR muestra característicamente disociación albúmino-ci- síndrome de Guillain-Barré?:
tológica: aumento de proteínas (a expensas de la albúmina) 1. Debilidad muscular progresivamente ascendente.
con número de células, normal. Esta alteración está presente 2. Enlentecimiento de la velocidad de conducción nerviosa en el
en un 80-90% de casos, pero suele aparecer a los 7-10 días estudio del nervio.
del comienzo de la enfermedad. 3. Disociación albúmino-citológica en el LCR.
Los estudios electrofisiológicos muestran enlentecimiento 4. Existencia de disautonomía.
precoz de las velocidades de conducción nerviosa. 5. Afectación de la musculatura respiratoria.

Tratamiento
78
Buen cuidado general y vigilancia ante posibilidad de afec- ¿Cuál de las siguientes entidades cursa con la tríada: ataxia cerebelosa,
tación de músculos respiratorios. Otros tratamientos emplea- retinitis pigmentaria y polineuropatía?:
dos han sido: Corticoides, Plasmaféresis e Inmunoglobulinas. 1. Leucodistrofia metacromática.
2. Enfermedad de Refsum.
Pronóstico 3. Enfermedad de Charcot Marie Tooth.
4. Enfermedad de Riley Day.
La mayoría se recuperan espontánea y completamente. Un 5. MERRF.
10-23 % requerirán ventilación mecánica. Un 7-22% quedan
con alguna secuela. La muerte se produce en un 2% y se debe
a desconexión accidental del respirador en una 1.a fase, y más 79
tarde a embolismo pulmonar como causa principal. ¿Cuál de los siguientes hallazgos no es típico de un síndrome de atrapa-
miento del nervio mediano?:
Neuropatía diabética 1. Debilidad para la extensión del primer dedo.
2. Debilidad para la oposición del pulgar.
Incluye los siguientes síndromes clínicos: 3. Alteración sensitiva en los 2/3 radiales de la palma de la mano.
4. Debilidad para la abducción del pulgar.
5. 1 y 3.
Oftalmoplejía diabética

Suele afectarse de forma aislada el III par; siendo el VI, el


segundo par más frecuentemente afectado. 80
Posiblemente se deben a infarto isquémico del nervio (afec- ¿Cuál de las siguientes entidades que causan polineuropatía produce tí-
tación de los vasa nervorum). El pronóstico es bueno. picamente enlentecimiento de la velocidad de conducción nerviosa?:
1. Porfiria.
Mononeuropatía aguda 2. Carcinoma.
3. Hipotiroidismo.
Los más frecuentemente afectados son N. Femoral y Ciático. 4. Tratamiento con vincristina.
5. Intoxicación con organofosforados.
Amiotrofia diabética
Debilidad y atrofia proximal, asimétrica y dolorosa, de curso RESPUESTAS: 76: 4; 77: 1; 78: 2; 79: 1; 80: 3.
subagudo.

1675
NEUROPATIAS PERIFERICAS

Polineuropatía distal, simétrica al dolor. Incluyen la PN sensitiva mutilante del adulto(HAD), si-
y de predominio sensitivo milar de la infancia(HAR) e Insensibilidad congénita al dolor.
Se afectan pies y piernas más que manos y brazos y su evo-
PN hereditarias con alteración metabólica conocida
lución es crónica y lentamente progresiva. Es la forma más fre-
cuente de Neuropatía diabética. Cuando se afecta sobre todo Enfermedad de Refsum
la sensibilidad profunda y la debilidad no es muy marcada, se
Retinitis pigmentaria, ataxia cerebelosa y polineuropatía
denomina seudotabes diabética por su parecido con la Tabes
dorsal. crónica junto con aumento del nivel sanguíneo de ácido fitáni-
co. La herencia es autosómica recesiva.
Neuropatía vegetativa Abetalipoproteinemia (S. Bassen-Kornzweig o Acantocitosis)
La herencia es autosómica recesiva (HAR). Presenta ausen-
Radiculopatía dolorosa toraco-abdominal cia de betalipoproteínas con nivel bajo de colesterol, degene-
ración retiniana macular, acantocitosis y déficit neurológico
Polineuropatía carcinomatosa y mielomatosa progresivo.
Es sensitiva o sensitivo-motora, simétrica, de predominio
distal y evolución lenta (la polineuropatía más frecuentemente Enfermedad de Tangier
asociada al carcinoma es axonal, subaguda o crónica y de pre- HAR. Hay deficiencia de lipoproteínas de alta densidad. La
dominio sensitivo). Es un efecto remoto del carcinoma, mielo- neuropatía es sensitivo-motora y asimétrica; puede aparecer
ma o, más raramente, del linfoma. El carcinoma de pulmón de forma recurrente, en ataques.
produce un 50% de los casos de neuropatía sensitiva pura;
también puede acompañar al plasmocitoma solitario del hue- E. Fabry
so. El origen es desconocido y el tratamiento consiste en el
control del crecimiento tumoral. La herencia es recesiva, ligada a X. Hay déficit de alfa-ga-
lactosidasa.
Neuropatías genéticamente determinadas
Leucodistrofia metacromática
Tipo mixto: sensitivomotor-vegetativo
HAR. Además del deterioro cerebral típico, se observa hipo-
Atrofia muscular peronea o enfermedad de Charcot-Marie- rreflexia, atrofia muscular y disminución de velocidad de con-
Tooth ducción nerviosa.
La herencia es autosómica dominante y comienza en la ado-
lescencia. Se caracteriza por pie en equinovaro, mano en garra Disautonomía familiar: Enf.de Riley-Day
y piernas muy delgadas (de cigüeña). El LCR es normal y están
disminuidas las velocidades de conducción nerviosa. HAR. Afecta sobre todo a niños judíos. Hay hiporreflexia,
Neuropatía hipertrófica progresiva o enfermedad de disminución de sensibilidad termoalgésica y preservación de
Déjerine-Sotas tacto-presión.
La herencia suele ser autosómica recesiva. Comienza en la
primera infancia y su evolución es lentamente progresiva. MONONEUROPATIA
También se puede observar pie en equinovaro y mano en ga-
rra; los nervios cubital, mediano, radial y peroneos están en- Se trata de un síndrome en el que la sintomatología está
grosados y no son dolorosos a la presión. Los enfermos quedan confinada al territorio de un solo nervio; puede estar afectado
confinados a una silla de ruedas a edad precoz. un plexo o parte del mismo. El término Causalgia se emplea
Distasia arrefléxica hereditaria o síndrome de Roussy-Lévy para un síndrome de dolor con sensación de quemazón que si-
Es una ataxia sensitiva con pies cavos y arreflexia. Se dis- gue a las lesiones de un nervio principal de la extremidad.
tingue de la Ataxia de Friedreich por el comienzo a edad tem- Meralgia parestésica: Afectación sensitiva desagradable en
prana, curso lento y benigno, ausencia de signos cerebelosos y cara anterolateral de muslo debida a compresión del N.
de tronco cerebral y preeminencia de la atrofia. El comienzo Femorocutáneo.
precoz y predominio de cuadro tabético lo distinguen de la
Atrofia muscular peronea.
MONONEUROPATIA MULTIPLE
Tipo predominantemente sensitivo
Se afectan varios nervios individuales al azar. Sus causas
Se caracterizan por dolores lancinantes, úlceras en manos y más frecuentes son panarteritis nodosa y diabetes; otras son
pies, osteomielitis de huesos de pies y manos e insensibilidad lepra y sarcoidosis.

1676
Capítulo XIV

SINDROMES
NEUROCUTANEOS
CONGENIT OS

Indice
Concepto Síndrome encefalo-trigeminal (enfermedad de
Neurofibromatosis (enfermedad de Von Reckling - Sturge-Weber)
hausen) Hemangioblastomatosis cerebelo-retiniana (síndro-
Esclerosis tuberosa (enfermedad de Bourneville) me de Von Hippel-Lindau)

CONCEPTO Anatomía patológica


Los neurofibromas son tumores que se originan de las célu-
Trastornos del desarrollo embrionario que afectan al siste-
ma nervioso y asocian alteraciones óseas y cutáneas. las de Schwann y de los fibroblastos.

NEUROFIBROMATOSIS (ENFERMEDAD DE Cuadro clínico


VON RECKLINGHAUSEN) — Tríada de Landowski:

Epidemiología • Neurofibromas de troncos nerviosos.


Dos formas transmisibles: • Neurofibromas cutáneos.
• Pigmentaciones cutáneas. Malignizan a sarcomas
— Tipo I: autosómica dominante con alteración en el cro- un 5-15%.
mosoma 17.
— Tipo II: autosómica dominante con alteración en el — Lesiones neurológicas:
cromosoma 22 que presenta tumores en vaina de ner-
vio acústico, generalmente bilateral y que no asocia • Neurofibromas de troncos nerviosos.
síntomas cutáneos. • Neurofibromas plexiformes.

1677
SINDROMES NEUROCUTANEOS CONGENITOS

• Otros tumores del SN: neurinoma del acústico, • Epilepsia: en los primeros años de vida. En oca-
meningioma, glioma del nervio óptico. siones las lesiones cutáneas ayudan al diagnósti-
• Ocasionalmente retraso mental. co de la epilepsia.

— Lesiones cutáneas: Retraso mental junto a epilepsia marcan un pobre pronósti-


co, muerte en la tercera década.
• Neurofibromas cutáneos, indoloros, muy numero-
sos, más en tronco. Pueden existir en el nacimiento. • Nódulos tumorales formados por neuronas y célu-
• Manchas café con leche: generalmente más de las gliales atípicas en corteza cerebral, ganglios
cinco, 1.5-5 cm., en tronco más frecuentes, bor- basales y próximos a ventrículo. Pueden producir
des nítidos, 10-20% de personas normales. hidrocefalia, pueden calcificarse.
• Efélides axilares .
• Nódulos de Lisch, hamartomas pigmentados del — Lesiones cutáneas.
iris.
• Adenoma sebáceo (angiofibroma), en forma de
— Lesiones óseas: alas de mariposa en mejillas, mentón y frente.
0.1-1 cm. 3-10 años.
• Cifoscoliosis. • Manchas blancas, son las primeras en aparecer.
• Ausencia congénita de esfenoides. Manchas hipopigmentadas en forma de hoja en
• Pseudoartrosis de tibia. tronco y extremidades.
• Fibromas periungueales (tumores de Koenen).
— Otros tumores: • Engrosamiento amarillento cutáneo en región
lumbosacra (piel de zapa, de cerdo o de tiburón).
• Feocromocitoma.
• Tumor de Wilms. — Otras lesiones:
• Rabdomiosarcoma.
• Neurinoma del VIII (bilateral en la forma tipo II). • Rabdomiomas cardíacos.
• Angioleiomiomas renales, hepáticos, suprarrena-
Tratamiento les y pancreáticos.
No existe.
Tratamiento
ESCLEROSIS TUBEROSA Sintomático.
(ENFERMEDAD DE BOURNEVILLE)
SINDROME ENCEFALO-TRIGEMINAL
Epidemiología (ENFERMEDAD DE STURGE-WEBER)
Puede heredarse de forma A.D. Alteración en el cromoso-
Epidemiología
ma 9.
Generalmente esporádica.
Anatomía patológica
Anatomía patológica
Las lesiones predominantes son los hamartomas.
Hemangiomas capilares o cavernosos junto con angiomas
Cuadro clínico predominantemente venosos en leptomeninges.
— Tríada clásica:
Cuadro clínico
• Ataques convulsivos. — Lesiones neurológicas: la corteza cerebral adyacente
• Retraso mental. al hemangioma se destruye de forma progresiva.
• Adenomas sebáceos.
• Crisis convulsivas. Es el primer síntoma neurológi-
Muerte por crisis convulsivas, tumores asociados o enfer- co. Crisis focal contralateral a la lesión cutánea.
medades intercurrentes. • Retraso mental.
• Secuelas de las crisis: parálisis sensitivo motora,
— Lesiones neurológicas. defectos campimétricos.

• Retraso mental. — Lesiones cutáneas:

1678
NEUROLOGIA

• Mancha “vino de Oporto”.


25
• Nevus vascular. 82
No es típico de la neurofibromatosis:
— Otras alteraciones:
1. Nódulos de Lisch.
• Glaucoma. 2. Asociación con patología tumoral.
3. Manchas «café con leche».
• Angiomas coroideos.
4. Presencia de retraso mental.
• Síndrome de Klippel-Trenaunay- Weber que aso- 5. Aparición de crisis convulsivas.
cia hemangioma en tronco o extremidades supe-
riores junto a malformación arteriovenosa de mé-
dula espinal y con hipertrofia de la extremidad
afecta.
82
— Calcificaciones “en raíles de tren” por depósito de Respecto a la esclerosis tuberosa es cierto que:
calcio en corteza cerebral. 1. Asocia alteraciones en el cromosoma 21.
2. Asocia retraso mental y crisis comiciales.
Pronóstico 3. Son típicos los nódulos de Lisch.
4. Frecuentemente aparece la mancha «vino de Oporto».
Supervivencia a largo plazo con trastronos mentales y he- 5. Se hereda de forma autosómica recesiva.
miparesia.

Tratamiento
Sintomático. 83
Respecto al síndrome de Von Hippel-Lindau es falso:
HEMANGIOBLASTOMATOSIS
CEREBELO-RETINIANA 1. Asocia angiomas y quistes hepáticos.
(SINDROME DE VON HIPPEL- LINDAU) 2. Herencia autosómica recesiva.
3. Existe una alteración en el cromosoma 3.
4. En el tratamiento puede ser útil la cirugía de los tumores cere-
Epidemiología belosos.
A.D. Alteración en cromosoma 3. 5. A veces asocia policitemia.

Cuadro clínico
Hemangioblastomas retinianos y cerebelosos. 84
— Lesiones neurológicas: hemangioblastomas cerebelo- Las calcificaciones «en raíles de tren» son típicas de:
sos, pueden ser múltiples, son quísticos, se encuen- 1. Neurofibromatosis.
tran en médula cerebelosa. También en bulbo o médu- 2. Enfermedad de los ferroviarios.
la espinal. Pueden causar hidrocefalia obstructiva. 3. Hemangioblastosis cerebelo-retiniana.
— Lesiones retinianas. Angiomas capilares que produ- 4. Síndrome de Sturge-Weber.
cen pérdida progresiva de visión. 5. Esclerosis tuberosa.
— Otras manifestaciones:

• Feocromocitoma.
• Policitemia (por producción ectópica de eritropo- 85
yetina). No es típico de la esclerosis tuberosa:
• Angiomas y quistes hepáticos pancreáticos y re-
nales. 1. Crisis comiciales.
2. Adenomas sebáceos.
3. Neurofibromas.
Tratamiento
4. Fibromas periungueales.
Fotocoagulación de lesiones retinianas. 5. Retraso mental.
Descompresión quirúrgica de tumores cerebelosos.

RESPUESTAS: 81: 5; 82: 2; 83: 2; 84: 4; 85: 3.

1679
Capítulo XV

PAT OLOGIA DE LA UNION


NEUROMUSCULAR
Indice
Trastornos autoinmunes Trastornos tóxicos
Trastornos congénitos

Dra. MARIA CASTRO DEL RIO

TRASTORNOS AUTOINMUNES Los anticuerpos (que son células T dependientes) se en-


cuentran en un 85% de los pacientes con Miastenia gene-
Miastenia gravis ralizada y en un 60% de aquellos con Miastenia ocular. El
nivel de los anticuerpos en sangre no se relaciona necesa-
Es un trastorno autoinmune raro, que se caracteriza por: de-
riamente con la severidad de la Miastenia. Los anticuerpos
bilidad fluctuante de ciertos músculos voluntarios (sobre todo,
bloquean la acción de la Acetilcolina liberada de las termi-
los inervados por nervios que dependen de los núcleos moto-
nales nerviosas.
res del tronco cerebral); debilidad manifiesta durante la activi-
dad continuada; rápida recuperación de la potencia con el re-
Clínica
poso y mejoría espectacular con fármacos anticolinesterásicos.
El comienzo suele ser insidioso, pero a veces es subagudo,
Epidemiología desencadenado por un trastorno emocional o por una infec-
ción. En ocasiones se manifiesta por primera vez durante el
La prevalencia es de 50-125/1.000.000; la proporción de
embarazo o puerperio o por el uso de ciertos fármacos durante
afectación mujer-hombre es 3:1 por debajo de los 40 años; pe-
la anestesia. Una vez que comienza, progresa lentamente,
ro por encima de esa edad, la incidencia en hombres es mayor.
afectando inicialmente a los músculos de los ojos. Pueden ocu-
Puede comenzar a cualquier edad, alcanzando un pico máximo
rrir remisiones durante un mes o dos (en menos de la 1/2 de
de comienzo a los 20-30 años.
casos). Si la enfermedad remite durante un año o más y des-
pués, recurre, tiende a ser progresiva.
Etiología y patogenia
Los músculos extraoculares se afectan en el 90%; su afecta-
Se produce una disminución de receptores disponibles ción es precoz y ocasiona los siguientes trastornos:
en la placa motora, que está causada por anticuerpos circu-
lantes de tipo IgG anti-receptor nicotínico de Acetilcolina — Diplopía binocular e intermitente (que se hace más
en la membrana postsináptica de la unión neuromuscular. manifiesta por las tardes).

1680
NEUROLOGIA

— Ptosis palpebral, que suele ser bilateral y agravada por


25
la luz solar; las respuestas pupilares son normales. 86
— También es común la debilidad de músculos faciales ¿Cuál de los siguientes hallazgos no esperaría encontrar en un paciente
(ej. cierre de la boca y cierre palpebral). diagnosticado de Miastenia gravis que presenta una crisis colinérgi-
— La afectación bulbar se suele manifestar por dificultad ca?:
para masticar y tragar, así como disartria, que es más
1. Debilidad muscular.
manifiesta después de estar hablando un tiempo.
2. Diarrea.
— La debilidad en las extremidades suele ser bilateral y 3. Palidez.
con afectación tanto proximal como distal. Los refle- 4. Midriasis.
jos osteotendinosos y la sensibilidad están preserva- 5. Náuseas y vómitos.
dos. No suele existir atrofia muscular.
Es característico que la debilidad se incremente a lo largo
87
del día; sólo hay un pequeño grupo de pacientes que se en-
cuentran peor tras despertarse por las mañanas. La debilidad ¿Cuál de los siguientes fármacos está contraindicado en la Miastenia
de músculos respiratorios sólo ocurre en casos avanzados. gravis?:
Existe una clasificación por estadíos clínicos de Osserman: 1. Neostigmina.
2. Neomicina.
III. Miastenia ocular. 3. Azatioprina.
III. A. Miastenia generalizada leve con progresión lenta; sin 4. Corticoides.
crisis y con respuesta farmacológica. 5. 2 y 3 están contraindicados.
III. B. Miastenia generalizada moderada; afectación severa
bulbar y esquelética; sin crisis y con respuesta menor a
fármacos. 88
III. Miastenia aguda fulminante: progresión rápida de síntomas ¿Cuál de las siguientes condiciones excluye el diagnóstico de Miastenia
severos, con crisis respiratorias y pobre respuesta a fárma- gravis?:
cos. Tiene alta incidencia de timomas y de mortalidad.
IV. Miastenia severa tardía: igual que III pero con progre- 1. Ausencia de detección de anticuerpos antirreceptor de acetilco-
lina en sangre.
sión en el curso de los años de I a II.
2. Ausencia de trastorno tímico.
3. Prueba del edrofonio negativa.
Trastornos asociados
4. Afectación de músculos de miembros.
Un 75 % de los pacientes tienen anomalías tímicas; de és- 5. Ninguna de las anteriores excluye el diagnóstico.
tos, un 85% tienen hiperplasia y un 15%, timomas.
En las mujeres jóvenes hay una estrecha relación con HLA-
B8 y DRW-3; siendo más alta la incidencia de HLA-B5 en muje- 89
res mayores.También hay una tasa más alta de anticuerpos en Uno de los siguientes hechos no es común a la Miastenia gravis y el sín-
los pacientes mayores.Se han descrito asociaciones con enfer- drome de Eaton-Lambert:
medades autoinmunes como L.E.S, Artritis reumatoide, 1. Debilidad muscular fluctuante.
Tiroiditis. En un 5% coinciden miastenia y tirotoxicosis. 2. Afectación del sistema nervioso autónomo.
3. Ambos son trastornos autoinmunes.
Diagnóstico 4. En ambos casos se pueden emplear corticoides.
Se realiza a partir de la clínica y las siguientes pruebas: 5. No hay afectación de la sensibilidad.

— EMG: muestra reducción de amplitud de los potencia-


les de acción del músculo tras estimulación repetida 90
de un nervio periférico.
— Anticuerpos anti-receptor de Ach: se encuentran en ¿Qué características de la oftalmopatía hacen sospechar una tirotoxicosis
más que una Miastenia gravis?:
más del 85% de los pacientes y son casi específicos
de la enfermedad. 1. Presencia de exoftalmos.
— Prueba del edrofonio: se inyectan 10 mg. (1 ml.) del 2. Ausencia de respuesta a neostigmina.
fármaco por vía i.v. (primero, 2 mg. y, si se toleran bien, 3. Buena respuesta a corticoides.
los restantes 8 mg., 30 segundos después). El efecto 4. Afectación del recto superior.
5. 1 y 2.
clínico se obtiene a los 30-60 segundos y dura 4-5 mi-
nutos. La positividad consiste en una mejoría objetiva y
subjetiva de la contractilidad muscular. El sulfato de RESPUESTAS: 86: 4; 87: 2; 88: 5; 89: 2; 90: 5.
atropina (0.6 mg.) permite contrarrestar su acción.

1681
PATOLOGIA DE LA UNION NEUROMUSCULAR

Esta prueba permite discernir entre crisis miasténicas Timectomía: se recomienda en prácticamente todos los ca-
y crisis colinérgicas ante una debilidad creciente; en sos de timoma. Las remisiones son de un 40-50% en los dos
el primer caso se produce mejoría, mientras en el se- primeros años de enfermedad.
gundo, no sólo no mejora, sino que hay un empeora- Plasmaféresis e inmunosupresión (Azatioprina).
miento de la debilidad.
— Rx de tórax y CT torácico: permite visualizar anoma- Síndrome de Eaton-Lambert
lías tímicas como el timoma.
Es un trastorno autoinmune de la unión neuromuscular, de-
bido a anticuerpos Ig G dirigidos contra los canales de calcio
Tratamiento
de las terminales presinápticas, con lo que se inhibe el flujo de
Fármacos anticolinesterásicos: Neostigmina (7.5-45 mg./2-6 calcio que facilita la difusión de la Acetilcolina hacia la mem-
horas) y Piridostigmina (dosis doble). Puede aparecer una crisis brana postsináptica.
colinérgica cuando se emplean dosis altas del fármaco. Los efec- El comienzo de la enfermedad tiene un pico de edad en tor-
tos muscarínicos (náuseas, vómitos, palidez, sudoración, diarrea, no a 50-60 años y están más afectados los varones (5:1).
miosis, bradicardia) que aparecen en la persona intoxicada se Suele debutar con debilidad de los músculos de las extremida-
acompañan de debilidad miasténica. No debe permitirse una des, afectando predominantemente la musculatura proximal; la de-
constricción pupilar por debajo de 2 mm. ni un descenso de la TA; bilidad de los músculos extraoculares y de control bulbar es inusual.
en esos casos, habrá que administrar Atropina i.v. Sólo si no se La potencia muscular puede mejorar transitoriamente tras las pri-
presentan los efectos muscarínicos y se sospecha una sobredosis meras contracciones musculares, pero rápidamente se sigue de fa-
de Neostigmina puede realizarse la prueba del Edrofonio. tigabilidad intensa. Los reflejos miotácticos están disminuidos, pero
Corticoides: se suelen emplear en los moderadamente gra- su abolición debe hacer pensar en una polineuropatía asociada.
ves que no responden a otros fármacos. Se utiliza Prednisona Se asocian trastornos autonómicos, como sequedad de bo-
en dosis progresivamente ascendentes. ca, estreñimiento, impotencia, anhidrosis, ojos secos...

TABLA II
Diagnóstico diferencial entre M.G. y S. Eaton-Lambert

Miastenia gravis S. Eaton-Lambert

Etiopatogenia Ac. contra receptor de Ach Ac. contra canales


postsináptico de Ca++ presináptico
Epidemiología Comienzo: 20-30 años Comienzo: >40 años
Mujeres (edad joven) Varones
Músculos afectados M. extraoculares y M. proximales de
principalmente bulbares extremidades
Reflejos OT Normales Disminuidos
S. autonómicos Ausentes Presentes
Mejoría Reposo Al comienzo del ejercicio
Anticolinesterásico. Guanidina
Empeoramiento Ejercicio Tubocurarina
EMG: Estím. aislado Potencial normal Pot. de baja ampl.
Estímulo repetido Respuesta decrec. Respuesta crec.
Asociaciones Alterac. tímicas Carcinomas (sobre
Enf. autoinmunes todo Oat-Cell).
Tratamiento Neostigmina Corticoides

1682
NEUROLOGIA

En un 50% de casos se encuentra un tumor asociado, que


25
suele ser Oat-cell. 91
En los estudios de electrodiagnóstico, un estímulo aislado Una de las siguientes medidas no debe ser empleada en un paciente con
del nervio puede dar lugar a un potencial de acción del múscu- Miastenia gravis:
lo de baja amplitud (que es normal en la M.G) y la estimulación
repetida da lugar a un marcado incremento de la amplitud (res- 1. Atropina.
2. Neostigmina.
puesta creciente). Los registros de fibra aislada muestran un
3. Timectomía.
incremento de la “tremulación”: Jitter. 4. Corticoides.
La respuesta a Neostigmina y Piridostigmina es variable. Se 5. Piridostigmina.
utiliza habitualmente como tratamiento prednisona o predniso-
na+Azatioprina (tabla II).
92
TRASTORNOS CONGENITOS Ante un paciente que está en estudio por un cuadro de debilidad muscu-
lar y el estudio eléctrico muestra un aumento progresivo de la ampli-
Miastenia transitoria del neonato tud de los potenciales de acción del músculo en respuesta a la esti-
Se debe al paso de Anticuerpos anti-receptor de Ach de la mulación repetida de un nervio periférico, ¿qué sospecharía usted?:
madre con Miastenia gravis al hijo, a través de la placenta. 1. Enfermedad de neurona motora.
2. Miastenia gravis.
Síndromes miasténicos congénitos 3. Síndrome de Eaton-Lambert.
Deficiencia de acetilcolinesterasa en la placa motora 4. Miositis por cuerpos de inclusión.
5. Polimiositis.
Se transmite con herencia autosómica recesiva.

Síndrome del canal lento 93


¿Cuál de los siguientes agentes actúa postsinápticamente en la produc-
Se debe a tiempo de apertura prolongado del canal de iones
ción del síndrome miasténico?:
inducido por Ach. Su herencia es autosómica dominante.
1. Anticuerpos anti-canales de calcio.
Miastenia infantil con HAR 2. Toxina botulínica.
3. Tetraciclinas.
Que se caracteriza por vesículas sinápticas muy pequeñas y 4. Neomicina.
los potenciales de placa terminal (MEPPPS) decrecen progresi- 5. Todos ellos actúan presinápticamente.
vamente con la actividad.

Deficiencia de receptores de Ach con HAR 94


Los varones mayores que padecen Miastenia gravis:
Un síndrome de debilidad y fatigabilidad de todos los mús-
culos, debido a “pobreza” congénita de vesículas sinápticas y 1. Tienen mayor incidencia de tumores tímicos.
de Ach liberada. 2. La debilidad muscular tiende a ser más generalizada que en las
mujeres jóvenes.
Síndrome de conductancia alta de canal rápido (HAD) 3. Se suele encontrar un título menor de anticuerpos antirreceptor
que en mujeres jóvenes.
Síndrome debido a interacción anormal de Ach-receptor 4. Son correctas 1 y 2.
5. Todas son correctas.
TRASTORNOS TOXICOS

Botulismo 95
La toxina actúa a nivel presináptico dificultando la difusión A un paciente con Miastenia generalizada moderada, sin crisis y con res-
puesta moderadamente satisfactoria a anticolinesterásicos, ¿qué
de Ach.
estadio clínico le correspondería?:
Fármacos 1. I.
2. IIa.
Existe una relación de más de 30 fármacos de uso habitual
3. IV.
que pueden interferir con la transmisión neuromuscular: hay 18 4. IIb.
antibióticos descritos, de los cuales destacan: neomicina, ka- 5. III.
namicina, estreptomicina y ciertas tetraciclinas (interfieren el
flujo de calcio, por lo que alteran la liberación del transmisor).
RESPUESTAS: 91: 1; 92: 3; 93: 5; 94: 1; 95: 4.

1683
Capítulo XVI

MIOPATIAS

Indice
Manifestaciones clínicas Clasificación de miopatias
Métodos diagnósticos

Dra. MARIA CASTRO DEL RIO

MANIFESTACIONES CLINICAS METODOS DIAGNOSTICOS

— Debilidad y pérdida de masa muscular: suele ser de E.M.G. y estudios de conducción nerviosa
predominio proximal y simétrica.
Los potenciales de acción de la unidad motora son polifási-
— Pseudohipertrofia muscular.
— Dolor muscular: cos y reducidos en amplitud y duración en las miopatías. En re-
poso, el músculo es eléctricamente silente, salvo en la poli-
• En reposo: puede ocurrir en las miopatías infla- miositis. El EMG puede ser normal, incluso cuando es obvia la
matorias, metabólicas, mioglobinúricas, mixede- debilidad.
matosa y algunas inducidas por fármacos.
• Durante o después del ejercicio: suele ocurrir en Enzimas musculares
polimiositis y miopatías metabólicas. Puede haber elevación en suero de aldolasa, GOT, LDH y
Creatín-kinasa, que es la más útil (normal<200 UI/l). Hay una
— Miotonía: es el retraso en la relajación tras una con- elevación muy marcada de CK en las distrofias de Duchenne y
tracción muscular voluntaria fuerte. El EMG es carac- Becker y en las miopatías inflamatorias. La CK se puede elevar
terístico. en otras circunstancias, como el ejercicio vigoroso, inyecciones
— Reflejos miotácticos: normalmente están preservados i.m., después de realizar un EMG, después de crisis tonico-cló-
hasta que la debilidad muscular es muy manifiesta nicas y de biopsia muscular.
(salvo en las distrofias).
— Calambres: Pueden ser causados por ejercicio inhabi- Biopsia muscular
tual, deshidratación y depleción de Na, hipomagnese-
mia, fallo renal, hipotiroidismo, hiopocalcemia y algu- Es útil y se debe realizar sobre un músculo moderadamente
nos fármacos: fenotiazinas, vincristina, salbutamol, li- afecto; nunca se realizará sobre músculos examinados previa-
tio... mente en un EMG.

1684
NEUROLOGIA

Test isquémico de lactato


25
96
Se utiliza cuando se sospecha déficit de alguna enzima gli-
Las citopatías mitocondriales:
colítica.
1. Sólo cursan con miopatía.
Test de lactato durante el ejercicio 2. Se heredan con carácter autosómico dominante.
3. Sólo se manifiestan clínicamente en las mujeres.
Una elevación anormal (>10 mmol/l.) en dicho test sugiere 4. Microscópicamente cursan con fibras rojas rasgadas.
una citopatía mitocondrial. 5. Siempre cursan con crisis epilépticas.

CLASIFICACION DE MIOPATIAS
97
Distrofias musculares ¿Cuál de las siguientes enfermedades no cursa con afectación de la moti-
lidad ocular extrínseca?:
Son trastornos hereditarios caracterizados por degeneración 1. Esclerosis lateral amiotrófica.
de la fibra muscular. 2. Miastenia gravis.
3. Síndrome de Kearns Sayre.
4. Miopatía de von Graefe Fuchs.
Distrofia de Duchenne
5. Miopatía hipertiroidea.
Comienza en la primera infancia (siempre antes de los 6
años). La herencia es recesiva ligada a X. La incidencia es de
13 a 33/100.000/año. 98
La forma más frecuente es la pseudohipertrófica, que se ca-
¿Cuál de las siguientes enfermedades musculares comienza a manifes-
racteriza por aumento anormal de tamaño de las pantorrillas y
tarse en una edad más temprana?:
otros músculos. Los primeros músculos afectados son los pel-
vicrurales y, más tarde, los de la cintura escapular. Las asocia- 1. Distrofia de Becker.
ciones más frecuentes son con retraso mental y con afecta- 2. Distrofia facioescapulohumeral.
ción cardíaca. 3. Distrofia miotónica de Steinert.
4. Distrofia de Emery-Dreiffus.
Los niveles de CK están muy elevados y la biopsia muscular
5. Distrofia de Duchenne.
muestra degeneración de fibras musculares, necrosis y susti-
tución por tejido fibroso. La muerte sobreviene en la adoles-
cencia habitualmente.
99
Distrofia de Becker Uno de los siguientes trastornos no es un síndrome paraneoplásico:
1. Dermatomiositis.
Es similar al tipo Duchenne, pero comienza más tarde, a 2. Síndrome de Eaton Lambert.
la edad de 10 años, es más benigna; el patrón de afecta- 3. Encefalitis límbica.
ción muscular también es similar al Duchenne. Es poco fre- 4. Enfermedad del central core.
cuente la asociación con retraso mental y el compromiso 5. Polineuropatía sensitiva pura.
cardíaco.
El nivel de la CK está moderadamente elevado y la herencia
es recesiva y ligada a X.
La ceguera para los colores se asocia al Becker y no al 100
Duchenne. Una de las siguientes afirmaciones sobre la miopatía por corticoides es
incorrecta:

Distrofia de las cinturas de los miembros 1. El EMG muestra potenciales de acción pequeños.
2. Se afecta la musculatura proximal de miembros y cinturas.
Comienza en la juventud temprana, en torno a los 20 años. 3. Los niveles de CPK y aldolasa están muy elevados.
La herencia es variable (recesiva o dominante). 4. La correlación entre dosis total de esteroides y gravedad clínica
es escasa.
Se afectan fundamentalmente los músculos de las cinturas 5. La supresión de los corticoides produce mejoría clínica.
escapular y pélvica y la muerte se suele producir alrededor de
los 50 años. Clínicamente es difícil de distinguir de la atrofia
muscular espinal. No suele haber deterioro intelectual ni com- RESPUESTAS: 96: 4; 97: 1; 98: 5; 99: 4; 100: 3.
promiso cardíaco.

1685
MIOPATIAS

Miopatía ocular de Von Graefe-Fuchs tosómica dominante. Afecta a músculos oculares y, más tar-
(oftalmoplejía externa progresiva) díamente, a faciales. La ausencia de retinitis pigmentaria y la
normalidad de crecimiento e intelecto la distinguen del síndro-
Comienza en la infancia o adolescencia. La herencia es au-
me de Kearns-Sayre.

TABLA III
Otras distrofias

HERENCIA EDAD COM. INICIO ASOCIAC.

D.F-E-H H.A.D 20 años Cintura


escapular

D.H-P H.A.D o Rx 20 años M. peroneos

D.E-H H.A.D Infancia o M. escápulo Cardio-


vida adulta humerales miopatía

D.M.C H.A.D o R Nacimiento Cinturas Retraso


escapular y mental y
pélvica Artrogriposis

D.F-E-H: Distrofia facioescapulohumeral.


D.H-P: Distrofia humeroperoneal o de Emery-Dreiffus.
D.E-H: Distrofia escápulo-humeral de Seitz.
D.M.C: Distrofia muscular congénita.
H.A.D: Herencia autosómica dominante.H.A.R: herencia autosómica recesiva. Rx: recesiva y ligada
a X.

OTRAS DISTROFIAS: ver tabla III: Se asocia con: cataratas, trastornos de conducción cardíaca,
atrofia gonadal, bajo coeficiente intelectual y en ocasiones,
Trastornos miotónicos demencia progresiva, somnolencia, hipoventilación pulmonar,
Distrofia miotónica de Steinert trastornos endocrinos, IgG reducida en suero e hipertermia ma-
ligna.
Es un trastorno con herencia autosómica dominante, que
afecta a otros órganos además del músculo. La incidencia es El EMG es característico y la biopsia muestra atrofia de fi-
de 14/100.000. Es la segunda distrofia más frecuente tras el bras de tipo I.
Duchenne.
Comienza al principio de la vida adulta y su curso es lenta- Miotonía congénita
mente progresivo. Se caracteriza por debilidad que afecta a los
El tipo que se presenta en la infancia, con herencia autosó-
músculos de las extremidades con un predominio distal; ade-
más, los músculos afectados presentan miotonía. Se produce mica dominante, es la enfermedad de Thomsen. Hay formas de
también debilidad de los músculos faciales, mandibulares y es- comienzo en la niñez con herencia autosómica recesiva y mio-
ternocleidomastoideos. Puede haber disartria, disfagia y pto- tonía generalizada que empeora con el frío y mejora con el ca-
sis. lor y el ejercicio.

1686
NEUROLOGIA

Miotonía neonatal
25
Ocurre en niños que nacen de madres miotónicas. 101
En el tratamiento de la miotonía se han utilizado procaina- Los ganglios basales comprenden los siguientes, excepto:
mida, difenilhidantoína, ACTH y corticoides. 1. Caudado.
2. Putamen.
Miopatías congénitas de cambios estructurales 3. Sustancia nigra.
en las fibras musculares 4. Núcleo subtalámico.
5. Amígdala.
Son trastornos raros que se manifiestan en la infancia con
hipotonía y debilidad de musculatura proximal. El diagnóstico
depende de la biopsia muscular.

Enfermedad del “Central Core ”(HAD) 102


El tratamiento de elección en los brotes de esclerosis múltiple es:
Hay una zona alterada en las porciones centrales de cada fi-
bra muscular que se tiñe con el P.A.S. 1. Corticoides.
2. Inmunosupresores.
3. Interferon.
Miopatía nemalínica (HAD) 4. Heparina.
Con la tinción de Gomori se observan miríadas de bastones 5. Inmunoglobulinas.
baciliformes, individuales y en grupos.

Miopatia centronuclear
103
Desproporción del tipo de fibras
En qué grupo de edades aparecen las convulsiones febriles:
Se han descrito fibras de tipo I más pequeñas y aumentadas
en número. 1. 3 años-5 años.
2. 3 meses-5 años.
3. 6 meses-3 años.
Miopatías metabólicas 4. 1 año-3 años.
Son trastornos hereditarios raros. 5. 1 mes-6 meses.

Citopatías mitocondriales
Se deben a defectos bioquímicos en la cadena respiratoria
104
de la mitocondria. Se han descrito los síndromes de:
La Miastenia gravis es un proceso producido por alteración a nivel de:
— MELAS (miopatía mitocondrial, encefalopatía, acido- 1. Asta anterior medular.
sis láctica y episodios similares a ACV) 2. Vía piramidal.
— MERRF (epilepsia mioclónica y fibras rojas melladas). 3. Placa motora.
— Recientemente se están incluyendo en este grupo, 4. Raíz anterior.
otras entidades como el S. de Kearns-Sayre. 5. Córtex motor.

Parálisis periódicas
Son ataques de debilidad muscular de las extremidades y el
tronco que suelen ocurrir después del ejercicio y pueden durar 105
de minutos a días. Diagnóstico de sospecha ante dolor orbitario con ptosis palpebral, miosis,
Tipos: enrojecimiento ocular y rinorrea de 60 a 180 minutos de duración:

— Hipokaliémica: los ataques suelen producirse durante 1. Migraña con aura visual.
el sueño, tras un día de ejercicio extenuante, y se ven 2. Migraña oftalmopléjica.
3. Glaucoma agudo.
favorecidos por las comidas ricas en carbohidratos. 4. Cefalea tensional aguda.
Durante los ataques, el nivel de potasio disminuye por 5. Cefalea en racimos.
debajo de 3 mmol/l.
— Hiperkaliémica: los ataques suelen desencadenarse por
el frío y el embarazo. Se alcanzan niveles de K>5 mmol/l. RESPUESTAS: 101:5; 102: 1; 103:2; 104: 3; 105: 5.
— Normokaliémica: es desencadenado por el frío, el al-

1687
MIOPATIAS

cohol y la sobrecarga de K. Miopatías endocrinas


— El hipertiroidismo se puede asociar a miopatía, paráli-
Las tres tienen una herencia A.D.
sis periódica y oftalmoplejía exoftálmica.
Miopatías mioglobinúricas — El hipotiroidismo puede producir calambres, debilidad
muscular e hiporreflexia.
Hay daño muscular severo con liberación de mioglobina a la
sangre y a la orina. Las causas son metabólicas, isquémicas, Otros trastornos que pueden producir debilidad muscular
inflamatorias y farmacológicas. son: Acromegalia, S. Cushing, E. Addison...
Miopatías inflamatorias
Miopatías inducidas por tóxicos y fármacos
Polimiositis/dermatomiositis
En un 25% de casos se asocian a enfermedad maligna o a Se han descrito:
una colagenosis.
Miopatía proximal dolorosa aguda o subaguda
Otros Betabloqueantes, cimetidina, colchicina, diacepam, isoniazi-
Miositis viral, Sarcoidosis, Miositis tropical... da, litio, vincristina, penicilina, fenitoína, teofilina...

1688
Capítulo XVII

DEGENERACIONES
ESPINOCEREBELOSAS

Indice
Concepto Ataxia de Friedrech
Clasificación Paraplejía espástica hereditaria

CONCEPTO ATAXIA DE FRIEDRECH

Síndromes caracterizados por ataxia y degeneración espino- La más frecuente. A.R. Alteración en cromosoma 9.
cerebelosa, ocasionalmente paraparesia espástica.
Anatomía patológica
CLASIFICACION
Degeneración neuronal crónica que afecta ganglios raquídeos
posteriores, columnas de Clarke y núcleo cuneiforme con dege-
Formas predominantemente espinales
neración de haces espinocerebelosos y neuronas piramidales
— Enfermedad de Friedrech. de la corteza motora.
— Distasia arrefléxica de Roussy- Lewy. Miocarditis intersticial.
— Ataxia cordonal posterior de Biemond.
Cuadro clínico
Formas espinocerebelosas
Debuta antes de los 20 años con ataxia de la marcha.
— Ataxia hereditaria tipo Menzel.
— Degeneración espinocerebelosa subaguda.
— Manifestaciones neurológicas: ataxia cerebelosa, ROT
abolidos, hipopalestesia en las cuatro extremidades,
Formas predominantemente cerebelosas
amiotrofia, sordera, atrofia óptica...
— Atrofia dentorrúbrica o disinergia cerebelosa miocló- — Manifestaciones esqueléticas: escoliosis, pie cavo.
nica de Ramsay- Hunt. — Manifestaciones cardíacas: miocardiopatías (más fre-
— Ataxia hereditaria tipo Holmes (atrofia cerebelo- oli- cuente la hipertrófica obstructiva).
var). — Manifestaciones endocrinas: diabetes mellitus.

1689
DEGENERACIONES ESPINOCEREBELOSAS

El paciente pierde la movilidad antes de los 40 años, el pro- Dos formas clínicas:
nóstico vital marcado por las alteraciones cardíacas.
— Tipo I en menores de 35 años.
PARAPLEJIA ESPASTICA HEREDITARIA — Tipo II que aparece en mayores de 35 años.

A.D. (A.R., ligada al sexo). La clínica es de paraparesia espástica. La hipertonía es ma-


Degeneración de haces piramidales distalmente y también yor en actividad. Evolución lenta. No modifica esperanza de
de cordones posteriores. vida.

BIBLIOGRAFIA

ADAMS, R.; VICTOR, M. Y COLS.: «Principles of Neurology». HARRISON Y COLS.: «Principios de Medicina Interna». 13.a edi-
Fifth edition. Mc Graw- Hill. 1993. ción. Interamericana, Mc Graw-Hill. 1994.
BALADRON, J.; VILLACAMPA, T.; VEGA, D. Y COLS.: «Manual in- PÉREZ, A.; MARTINEZ B. Y COLS.: «Manual de urgencias neuroló-
tensivo para el examen MIR». Luzán 5. 1993. gicas». Ergón. 1993.
GIMENO, A. Y COLS.: «Medicine. Tratado de Medicina Interna».
Neurología. Sexta edición. IDEPSA. 1994.

1690
Sección 25

INDICE
DE
MATERIAS

Accidente cerebrovascular, 1640 Negligencia, 1630


Accidente isquémico transitorio, 1640 neuritis óptica, 1660
afasia, 1631 Neurosífilis, 1650
agonistas dopaminérgicos, 1644 Nódulos de Lisch, 1678
Anillo de Kayser-Flescher, 1646 Panencefalitis esclerosante subaguda (PEES), 1653
Cefalea histamínica, 1668 paratonía, 1629
ceguera cortical, 1630 Reflejos óculo-cefálicos, 1634
Ceruloplasmina, 1646 Reflejos óculo-vestibulares, 1634
Coma, 1633 Seudodemencia, 1639
decorticación, 1634 Signo de Lhermitte, 1660
degeneración subaguda combinada de la médula, 1672 Síndrome,
Demencia, 1636 Brown-Sequard, 1672
descerebración, 1634 de Balint, 1630
E. Creutzfeldt-Jakob, 1653 de Eaton-Lambert, 1682
enfermedad de la neurona motora, 1657 de Gerstmann, 1630
Enfermedad de Lyme, 1651 de Guillain-Barré, 1674
escala de Glasgow, 1634 de Klippel-Trenaunay, 1679
Ganglios basales, 1643 de Lennox-Gastaut, 1664
Herniación uncal, 1634 de Miller-Fisher, 1675
Ictus lacunar, 1641 de West, 1665
Leucoencefalopatía multifocal progresiva (LMP), 1652 Hakim-Adams, 1638
levodopa, 1644 Status epiléptico, 1665
Meningitis aséptica, 1652 tabes dorsal, 1672
migraña basilar (Bickerstaff), 1668 Tríada de Landowski, 1677
migraña oftalmopléjica, 1668 Tromboflebitis intracraneal, 1650
moria, 1630 tumores de Koenen, 1678

1691

Das könnte Ihnen auch gefallen